ΠΡΟΤΕΙΝΟΜΕΝΕΣ ΑΣΚΗΣΕΙΣ ΓΙΑ ΜΑΘΗΤΙΚΟΥΣ ΔΙΑΓΩΝΙΣΜΟΥΣ 201...

89
http://www.mathematica.gr/forum/viewtopic.php?f=109&t=15584 Επιμέλεια: xr.tsif Σελίδα 1

Transcript of ΠΡΟΤΕΙΝΟΜΕΝΕΣ ΑΣΚΗΣΕΙΣ ΓΙΑ ΜΑΘΗΤΙΚΟΥΣ ΔΙΑΓΩΝΙΣΜΟΥΣ 201...

Page 1: ΠΡΟΤΕΙΝΟΜΕΝΕΣ ΑΣΚΗΣΕΙΣ ΓΙΑ ΜΑΘΗΤΙΚΟΥΣ ΔΙΑΓΩΝΙΣΜΟΥΣ 201 - 300

http://www.mathematica.gr/forum/viewtopic.php?f=109&t=15584

Επιμέλεια: xr.tsif Σελίδα 1

Page 2: ΠΡΟΤΕΙΝΟΜΕΝΕΣ ΑΣΚΗΣΕΙΣ ΓΙΑ ΜΑΘΗΤΙΚΟΥΣ ΔΙΑΓΩΝΙΣΜΟΥΣ 201 - 300

http://www.mathematica.gr/forum/viewtopic.php?f=109&t=15584

Επιμέλεια: xr.tsif Σελίδα 2

ΠΡΟΤΕΙΝΟΜΕΝΕΣ

ΑΣΚΗΣΕΙΣ ΓΙΑ ΜΑΘΗΤΙΚΟΥΣ

ΔΙΑΓΩΝΙΣΜΟΥΣ

ΤΕΥΧΟΣ 3ο

ΑΣΚΗΣΕΙΣ 201 - 300

Αφιερωμένο σε κάθε μαθητή που ασχολείται ή πρόκειται να ασχοληθεί με

Μαθηματικούς διαγωνισμούς

Τσιφάκης Χρήστος : xr.tsif

Page 3: ΠΡΟΤΕΙΝΟΜΕΝΕΣ ΑΣΚΗΣΕΙΣ ΓΙΑ ΜΑΘΗΤΙΚΟΥΣ ΔΙΑΓΩΝΙΣΜΟΥΣ 201 - 300

http://www.mathematica.gr/forum/viewtopic.php?f=109&t=15584

Επιμέλεια: xr.tsif Σελίδα 3

ΘΕΜΑ 201 (Socrates)

Αν x,y,z 0 να δείξετε ότι x y y z z x

x y z .y 2 z 2 x 2

Λύση:

Αρκεί να δείξουμε ότι x y

xy 2

,

y zy

z 2

,

z xz

x 2

, αφού τότε με πρόσθεση

κατά μέλη θα πάρουμε τη ζητούμενη.

Πράγματι

x yx xy 2x x y x y xy 0

y 2

y zy y z yz 0

z 2

z xz z x zx 0

x 2

, που ισχύουν.

ΘΕΜΑ 202 (Socrates)

Αν οι πραγματικοί αριθμοί a,b,c είναι τέτοιοι ώστε 2 2 2 3 3 3

a b c a b c 1 , να

βρείτε την τιμή της παράστασης 2 9 2011

P a b c .

(Από τα παραπάνω παραδείγματα, έχει γίνει φανερό το τι προσπαθούμε να

πετύχουμε για να λύνουμε τέτοιου είδους ασκήσεις)

Λύση:

(Α) Έστω 0 a,b,c 1 τότε

3 2a 1 a a

3 2b 1 b b

Page 4: ΠΡΟΤΕΙΝΟΜΕΝΕΣ ΑΣΚΗΣΕΙΣ ΓΙΑ ΜΑΘΗΤΙΚΟΥΣ ΔΙΑΓΩΝΙΣΜΟΥΣ 201 - 300

http://www.mathematica.gr/forum/viewtopic.php?f=109&t=15584

Επιμέλεια: xr.tsif Σελίδα 4

3 2c 1 c c

και με πρόσθεση κατά μέλη προκύπτει 3 3 3 2 2 2a b c a b c 1 1 άτοπο.

(Β) Έστω a,b,c 0 τότε 3 3 3a b c 0 1 0 άτοπο.

(Γ) Έστω ένα τουλάχιστον από τους αριθμούς a,b,c είναι μεγαλύτερο της μονάδας,

υποθέτουμε χωρίς βλάβη της γενικότητας ότι

a 1 τότε 3 2a a 1 όμως 2 2 2 2 2 2

a b c 1 a 1 b c .

Άρα, 2 2 2 2 2a 1 1 b c 1 b c 0 άτοπο.

(Δ) Έστω ένα τουλάχιστον από τους αριθμούς a,b,c είναι αρνητικός, υποθέτουμε

χωρίς βλάβη της γενικότητας ότι a 0 τότε πρέπει κάποιο από τα b,c να είναι

μεταξύ του μηδέν και ένα, (αν είναι μεγαλύτερο του ένα, τότε από την Γ περίπτωση

θα καταλήγαμε σε άτοπο, αν και οι δύο ήσαν αρνητικοί αριθμοί τότε από την Β

περίπτωση θα ήταν άτοπο), έστω 0 b 1 , τότε: 3a 0 και 2

a 0 άρα 3 2a a ,

επίσης 3 2b b και με πρόσθεση κατά μέλη παίρνουμε: 3 3 2 2

a b a b δηλαδή *c 0

3 2 2 31 c 1 c c c 1 c

που λόγω της Γ περίπτωσης είναι άτοπο.

* Θα δείξουμε ότι c 0 .

Έστω ότι c 0 , τότε η δεδομένη σχέση γίνεται 3 3 2 2

a b a b άτοπο γιατί έχουμε

αποδείξει ότι: 3 3 2 2

a b a b .

Σημείωση: Δεν χρειάζεται, αλλά το δίνω...Θα δείξουμε ότι c 0 .

3a 1 1

3 3 3 3 3 3 3 3 3 3 3a b c 1 a 1 b c 0 b c 1 c 1 b

3 2 3c 1 b 1 b b 0 c 0 c 0 .

και εδώ τελειώνει η διερεύνηση.

Page 5: ΠΡΟΤΕΙΝΟΜΕΝΕΣ ΑΣΚΗΣΕΙΣ ΓΙΑ ΜΑΘΗΤΙΚΟΥΣ ΔΙΑΓΩΝΙΣΜΟΥΣ 201 - 300

http://www.mathematica.gr/forum/viewtopic.php?f=109&t=15584

Επιμέλεια: xr.tsif Σελίδα 5

Οι περιπτώσεις όλοι οι αριθμοί να είναι μηδέν ή ένα , εύκολα απορρίπτονται και

μένει ένας από αυτούς να είναι 1 και οι άλλοι μηδέν, πχ. a 1 , b c 0 τότε 2 9 2011

P a c 1b .

Β τρόπος

Είναι 2 2 2 2a a b c 1 άρα 2

a 1 οπότε και a 1 . Όμοια b 1 , c 1 . Άρα

2 2 2 3 3 3 2 2 20 1 1 (a b c ) (a b c ) a (1 a) b (1 b) c (1 c) άθροισμα

θετικών.

Συνεπώς 2 2 20 a (1 a) b (1 b) c (1 c) δηλαδή τα a,b,c παίρνουν μόνο τις τιμές

0ή 1 . Με έλεγχο διαπιστώνουμε ότι οι μόνες αποδεκτές είναι οι

(1,0,0),(0,1,0),(0,0,1) και η τελική παράσταση είναι 1 σε όλες τις περιπτώσεις.

ΘΕΜΑ 203 (ΔΗΜΗΤΡΗΣ ΙΩΑΝΝΟΥ )

a) Να αποδειχθεί ότι υπάρχουν άπειρες τριάδες θετικών ακεραίων m,n,k έτσι, ώστε

να ισχύει ότι: 2

4mn m n k 1

b) Να αποδείξετε ότι δεν υπάρχουν θετικοί ακέραιοι m,n,k έτσι, ώστε να ισχύει ότι: 2

4mn m n k .

Λύση:

Π.χ. 2(m,n,k) (1,3 ,3 )

Είναι 22(4m4mn m 1)(4n 1)n k 4k 1 . Υπάρχει πρώτος p της μορφής

p 4a 3 τέτοιος ώστε p / 4m 1 ,οπότε και 2p / 4k 1 .

Αυτό όμως είναι άτοπο, δείτε π.χ.

www.mathematica.gr/forum/viewtopic.php?p=88420#p88420.

Page 6: ΠΡΟΤΕΙΝΟΜΕΝΕΣ ΑΣΚΗΣΕΙΣ ΓΙΑ ΜΑΘΗΤΙΚΟΥΣ ΔΙΑΓΩΝΙΣΜΟΥΣ 201 - 300

http://www.mathematica.gr/forum/viewtopic.php?f=109&t=15584

Επιμέλεια: xr.tsif Σελίδα 6

ΘΕΜΑ 204 (Socrates)

Να βρείτε τους πρώτους p,q,r έτσι ώστε οι αριθμοί pq qr rp και

3 3 3p q r 2pqr να διαιρούνται με τον p q r .

Λύση:

Έχουμε 2 2 2 3 3 3(p q r)(p q r pq pr qr) p q r 3pqr . Άρα p q r

διαιρεί το 2 2 2(p q r)(p q r pq pr qr) pqr και άρα διαιρεί το pqr .

Επειδή οι p,q,r είναι πρώτοι, χωρίς βλάβη της γενικότητας μπορούμε να υποθέσουμε

ότι ο pδιαιρεί τον pqr .

[Αυτό ισχύει διότι πρέπει ο pqr να ισούται με ένα από τους p,q,r,pq,pr,qr,pqr ].

Επειδή όμως ο pqr διαιρεί και τον pq qr rp και ο pδιαιρεί αυτόν τον αριθμό

και άρα διαιρεί και τον qr . Επειδή όμως ο p είναι πρώτος πρέπει να διαιρεί είτε τον

q είτε τον r , χωρίς βλάβη της γενικότητας τον q . Αλλά και ο q είναι πρώτος και

άρα p q .

Περίπτωση 1η:

Ο r διαιρεί τον p q r . Τότε (αφού ο p q r διαιρεί τον pq qr rp ) ο r

διαιρεί και το 2pq p και άρα r p . Σε αυτήν την περίπτωση οι συνθήκες γίνονται

33p / 3p (η οποία ισχύει για όλα τα p ) και 3

3p / p η οποία ισχύει αν και μόνο αν

p 3 .

Περίπτωση 2η:

Ο r δεν διαιρεί τον p q r . Τότε (αφού ο p q r διαιρεί τον 2pqr p r ) πρέπει

ο p q r να ισούται είτε με p είτε με 2p με το πρώτο να απορρίπτεται. Αλλά τότε

2r p 2p p(p 2) ο οποίος δεν είναι πρώτος εκτός από την περίπτωση που

r p 3 . (Οπότε αναγόμαστε πάλι στην 1η περίπτωση.)

Άρα τελικά η μοναδική λύση είναι η q r p 3 .

Page 7: ΠΡΟΤΕΙΝΟΜΕΝΕΣ ΑΣΚΗΣΕΙΣ ΓΙΑ ΜΑΘΗΤΙΚΟΥΣ ΔΙΑΓΩΝΙΣΜΟΥΣ 201 - 300

http://www.mathematica.gr/forum/viewtopic.php?f=109&t=15584

Επιμέλεια: xr.tsif Σελίδα 7

ΘΕΜΑ 205 (Socrates)

Πόσοι τετραψήφιοι αριθμοί abcd είναι τέτοιοι ώστε a b c d και 2 2 2 2

a b c d ;

Λύση:

Έχουμε a b c d (1), και 2 2 2 2a b c d (2)

Άρα 2 2(a b) 2ab (c d) 2cd 2ab 2cd (3)

Με πρόσθεση κατά μέλη των σχέσεων (2) και (3) παίρνουμε

2 2(a b) (c d) a b c d a b c d

1η περίπτωση: a b c d (4)

Τότε από τις σχέσεις (1),(4) έπεται ότι a c, b d και άρα ο τετραψήφιος αριθμός

είναι της μορφής abab

Και τέτοιας μορφής τετραψήφιοι είναι στο πλήθος 90 (αφού το a μπορεί να πάρει

τις τιμές 1,2,3,...,9 και το b τις τιμές 0,1,2,3,...,9 .

2η περίπτωση: a b c d (5)

Τότε από τις σχέσεις (1),(5) έπεται ότι a d, b c και άρα ο τετραψήφιος αριθμός

είναι της μορφής abba

Και τέτοιας μορφής τετραψήφιοι είναι στο πλήθος πάλι 90 . Όμως από αυτούς θα

πρέπει να αφαιρέσουμε όσοι ήδη υπάρχουν στην πρώτη περίπτωση δηλαδή όσοι

έχουν a b . Δηλαδή πρέπει να αφαιρέσουμε τους αριθμούς 1111,2222,...,9999 οι

οποίοι είναι 9 στο πλήθος.

Από τα παραπάνω, συμπεραίνουμε ότι όλοι οι ζητούμενοι τετραψήφιοι είναι

90 90 9 171 .

Page 8: ΠΡΟΤΕΙΝΟΜΕΝΕΣ ΑΣΚΗΣΕΙΣ ΓΙΑ ΜΑΘΗΤΙΚΟΥΣ ΔΙΑΓΩΝΙΣΜΟΥΣ 201 - 300

http://www.mathematica.gr/forum/viewtopic.php?f=109&t=15584

Επιμέλεια: xr.tsif Σελίδα 8

ΘΕΜΑ 206 (ΔΗΜΗΤΡΗΣ ΙΩΑΝΝΟΥ )

Να αναλυθεί ο αριθμός 20102 1 σε γινόμενο δύο παραγόντων, έτσι ώστε ο καθένας

να είναι μεγαλύτερος του 10042 .

Λύση:

2010 1005 503 1005 5032 1 2 1 2 2 1 2 A·B , όπου 1005 503 1004

A 2 1 2 2 και

1005 503 1004 503 1004 1004 503 1004B 2 1 2 2·2 2 1 2 2 2 1 2 .

ΘΕΜΑ 207 (Cretanman )

Να δείξετε ότι υπάρχουν τουλάχιστον 5 ζευγάρια από διαδοχικούς θετικούς

ακεραίους που είναι τέτοιοι ώστε το άθροισμα των τετραγώνων των αριθμών σε κάθε

ζευγάρι να διαιρεί τον αριθμό 20062 1 .

Λύση:

Προς το παρόν έχω βρει τέσσερα ζευγάρια. Δίνω το σκεπτικό: Αν x,x 1 είναι οι

δυο διαφορετικοί ακέραιοι, τότε 2 2 2x (x 1) 2x 2x 1 . Αυτό μοιάζει πολύ με

την παραγοντοποίηση 2006 1005 503 1005 5032 1 (2 2 1)(2 2 1) που έκανε ο

Αλέξανδρος στο πρόβλημα 206. Κάνοντας το ίδιο για το 2006

2 1 έχουμε 2006 1003 502 1003 502

2 1 (2 2 1)(2 2 1) . Τώρα παρατηρούμε ότι για 501x 2 έχουμε

2 1003 502 20062x 2x 1 2 2 1/ 2 1 .

Βρήκαμε λοιπόν το πρώτο ζευγάρι. Είναι το 501 501(2 ,2 1) .

Αφού αυτό δούλεψε ας προσπαθήσουμε να συνεχίσουμε με παρόμοιο τρόπο. Για

κάθε μη αρνητικό ακέραιο n έχουμε 4n 2 2n 1 n 1 2n 1 n 12 1 (2 2 1)(2 2 1)

.

Επίσης για n

x 2 έχουμε 2 2n 1 n 1

2x 2x 1 2 2 1 . Άρα αν

4n 2 20062 1 / 2 1

,

τότε παίρνουμε και το ζευγάρι n n(2 , 2 1) .

Μπορούμε να βρούμε τέτοιους αριθμούς;

Page 9: ΠΡΟΤΕΙΝΟΜΕΝΕΣ ΑΣΚΗΣΕΙΣ ΓΙΑ ΜΑΘΗΤΙΚΟΥΣ ΔΙΑΓΩΝΙΣΜΟΥΣ 201 - 300

http://www.mathematica.gr/forum/viewtopic.php?f=109&t=15584

Επιμέλεια: xr.tsif Σελίδα 9

Βασικό λήμμα: Αν a,b φυσικοί αριθμοί με b περιττό, τότε το a2 1 διαιρεί το

ab2 1 .

Απόδειξη: a a 2a 3a (b 1)a a 2a 3a (b 1)a(1 2 )(1 2 2 2 2 ) (1 2 2 2 2 )

a 2a 3a 4a ba ab(2 2 2 2 2 ) 2 1 .

[Η υπόθεση b περιττός χρειάζεται για να έχουμε πλην στον συντελεστή του (b 1)a2

στο αριστερό μέλος.]

Επειδή 2006 2 17 59 από το βασικό λήμμα συμπεραίνουμε ότι m 20062 1 | 2 1 για

κάθε m {2,2 17,2 59,2 17 59} {2,34,118,2006} . Επειδή όλοι αυτοί οι αριθμοί

είναι της μορφής 4n 2 παίρνουμε τα ζευγάρια 8 8 29 29 501 501

(1,2),(2 ,2 1),(2 ,2 1),(2 ,2 1) .

Μας λείπει ακόμη ένα ζευγάρι. Αν και ο Αλέξανδρος ζητάει θετικούς ακεραίους, θα

κλέψω και θα πω ότι είναι το (0,1) . Αν μας πει ότι θέλει ακόμη ένα θα πρέπει να

ξανασκεφτούμε τι να κάνουμε...

Είναι 1005 503 1004 502 22 2 1 2 (2 1) , οπότε άλλο ένα ζεύγος είναι

502 502(2 1,2 ).

Παρόμοια, μπορούμε να βρούμε και άλλα ζεύγη.

ΘΕΜΑ 208 (Socrates)

Οι θετικοί ακέραιοι a,b,n είναι τέτοιοι ώστε ο αριθμός 2 2

a 2nb να είναι τέλειο

τετράγωνο ακεραίου. Να δείξετε ότι ο αριθμός 2 2

a nb γράφεται ως άθροισμα δύο

τέλειων τετραγώνων.

Λύση:

Έχουμε: 2 2 2 2 2 2 2 2 2 2 2 2

a 2nb k a 2nb a k a 2a 2nb k b 2 2

2 2 2 2 2 2k a k a k aa nb a nb ( ) ( )

2 2 2

.

Page 10: ΠΡΟΤΕΙΝΟΜΕΝΕΣ ΑΣΚΗΣΕΙΣ ΓΙΑ ΜΑΘΗΤΙΚΟΥΣ ΔΙΑΓΩΝΙΣΜΟΥΣ 201 - 300

http://www.mathematica.gr/forum/viewtopic.php?f=109&t=15584

Επιμέλεια: xr.tsif Σελίδα 10

Μένει να αποδείξουμε ότι οι αριθμοί k a k a

,2 2

είναι ακέραιοι.

Πράγματι, αν a άρτιος, τότε επειδή 2 2 2a 2nb k θα πρέπει και ο k να είναι άρτιος

(γνωστό) οπότε και k a άρτιος και k a άρτιος και άρα οι αριθμοί k a k a

,2 2

είναι ακέραιοι.

Αν πάλι είναι a περιττός τότε πρέπει και ο k να είναι περιττός οπότε οι αριθμοί

k a και k a είναι άρτιοι και έχουμε και πάλι το ζητούμενο.

ΘΕΜΑ 209 (Socrates)

Αν S(n) το άθροισμα των ψηφίων του φυσικού n να δείξετε ότι ο αριθμός

2S(2n 3) δεν είναι τέλειο τετράγωνο ακεραίου.

Λύση:

Είναι γνωστό ότι ένας αριθμός n είναι ισότιμος mod9 με το άθροισμα των

ψηφίων του, δηλαδή 2 2S(2n 3) 2n 3 .Επομένως αρκεί να απο δείξουμε ότι ο

αριθμός 22n 3 δεν είναι τέλειο τετράγωνο.

Τα τετραγωνικά υπόλοιπα mod9 είναι τα 0,1,4,7 .

Άρα 2 22n 0,2,5,8(mod9) 2n 3 2,3,5,8(mod9) ,που δεν είναι τετραγωνικά

υπόλοιπα mod9 .

Συνεπώς ο αριθμός 2S(2n 3) δεν είναι τέλειο τετράγωνο.

ΘΕΜΑ 210 (Socrates)

Να βρείτε τους πρώτους p,q,r ώστε οι αριθμοί

2 2 2pq r,pq r ,qr p,qr p ,rp q,rp q να είναι επίσης πρώτοι.

Page 11: ΠΡΟΤΕΙΝΟΜΕΝΕΣ ΑΣΚΗΣΕΙΣ ΓΙΑ ΜΑΘΗΤΙΚΟΥΣ ΔΙΑΓΩΝΙΣΜΟΥΣ 201 - 300

http://www.mathematica.gr/forum/viewtopic.php?f=109&t=15584

Επιμέλεια: xr.tsif Σελίδα 11

Λύση:

Απάντηση: Μόνο η περίπτωση p 2,q 3,r 5 ή κυκλικά.

Λύση: Δεν μπορεί να είναι και οι τρεις από τους p,q,r περιττοί γιατί θα ήταν άρτιος

2 ο pq r . Άρα κάποιος είναι άρτιος πρώτος, και χωρίς βλάβη p 2 . Δηλαδή η

υπόθεση τώρα γίνεται "οι αριθμοί 2 22q r,2q r ,qr 2,qr 4,2r q,2r q είναι

όλοι πρώτοι".

Λόγω του qr 2 , οι q,r είναι και οι δύο περιττοί.

Χρησιμοποιούμε το γεγονός ότι οι περιττοί πρώτοι από τον 5 και πέρα είναι της

μορφής 6k 1 . Θα δούμε τώρα ότι δεν μπορεί και οι δύο από τους q,r να είναι 5 .

Πράγματι, αποκλείεται η περίπτωση q 6Q 1,r 6R 1 γιατί τότε

qr 2 (6Q 1)(6R 1) 2 6S 3 σύνθετος.

Όμοια αποκλείεται η περίπτωση q 6Q 1,r 6R 1 καθώς και η

q 6Q 1,r 6R 1 λόγω του qr 4 .

Επίσης αποκλείεται η q 6Q 1,r 6R 1 λόγω του qr 2 .

Συμπεραίνουμε ότι τουλάχιστον ένας από τους περιττούς πρώτους q,r είναι 5 .

Χωρίς βλάβη q 3 .

H υπόθεση τώρα γίνεται "οι αριθμοί 26 r,6 r ,3r 2,3r 4,2r 3,2r 9 είναι

όλοι πρώτοι". (*)

Με το χέρι ελέγχουμε ότι η r 5 δίνει λύση αλλά οι r 3,7,11,13,17,19,23,29 δεν

δίνουν λύση (άμεσο).

Θα δούμε ότι δεν έχουμε άλλη λύση. Οι πρώτοι που περισσεύουν είναι της μορφής

r 30k 1,30k 7,30k 11,30k 13,30k 17,30k 19,30k 23,30k 29 .

Ελέγχοντας με το χέρι διαπιστώνουμε ότι κανένας από αυτούς δεν δίνει όλους τους

(*) πρώτους.

Page 12: ΠΡΟΤΕΙΝΟΜΕΝΕΣ ΑΣΚΗΣΕΙΣ ΓΙΑ ΜΑΘΗΤΙΚΟΥΣ ΔΙΑΓΩΝΙΣΜΟΥΣ 201 - 300

http://www.mathematica.gr/forum/viewtopic.php?f=109&t=15584

Επιμέλεια: xr.tsif Σελίδα 12

Συγκεκριμένα, σε ΚΑΘΕ περίπτωση τουλάχιστον ένας από τους (*) βγαίνει

πολλαπλάσιο του 5 .

Π.χ. ο r 30k 1 δίνει πολλαπλάσιο του 5 τον 2r 3 , ο r 30k 7 δίνει

πολλαπλάσιο του 5 τον 3r 4 , και ούτω καθ' εξής (ρουτίνα).

Δείξαμε ότι οι μόνες αποδεκτές περιπτώσεις είναι οι παραπάνω p 2,q 3,r 5 ή

κυκλικά.

ΘΕΜΑ 211 (Socrates)

Να δείξετε ότι a(a 1) b(b 1) c(c 1) (a b c 4)(a b c 5 4 ) , αν

a,b,c 2.

Λύση:

Αρκεί να αποδείξουμε ότι:

2 2 2a b c (a b c) (a b c 4)(a b c 5) 4

2(a b c) 2(ab ac cb) (a b c) (a b c 4)(a b c 5) 4

Θέτουμε a b c t . Τότε αρκεί να αποδείξουμε ότι:

2t 2(ab ac bc) t (t 4)(t 5) 4

4t ab ac cb 12 4(a b c) ab ac cb 12 (1).

Θέτουμε x a 2,y b 2,z c 2 . Τότε από την υπόθεση έχουμε ότι

x,y,z 0 και η σχέση (1) ισοδύναμα γράφεται:

4(x 2 y 2 z 2) (x 2)(y 2) (x 2)(z 2) (y 2)(z 2) 12

0 xy xz zy η οποία είναι αληθής, αφού x,y,z 0 .

Page 13: ΠΡΟΤΕΙΝΟΜΕΝΕΣ ΑΣΚΗΣΕΙΣ ΓΙΑ ΜΑΘΗΤΙΚΟΥΣ ΔΙΑΓΩΝΙΣΜΟΥΣ 201 - 300

http://www.mathematica.gr/forum/viewtopic.php?f=109&t=15584

Επιμέλεια: xr.tsif Σελίδα 13

ΘΕΜΑ 212 (Socrates)

Θεωρούμε σκακιέρα 50x50 . Αρχικά όλα τα τετράγωνα 1x1έχουν μαύρο χρώμα. Μια

κίνηση συνίσταται στο να αλλάξουμε το χρώμα όλων των τετραγώνων μιας στήλης ή

μιας γραμμής (αν είναι μαύρο γίνεται άσπρο και αν είναι άσπρο γίνεται μαύρο.)

α) Δείξτε ότι δεν είναι δυνατόν, μετά από ένα αριθμό κινήσεων, να έχουμε ακριβώς

2011 άσπρα τετράγωνα στη σκακιέρα .

β) Μπορεί να προκύψει σκακιέρα με ακριβώς 2010 άσπρα τετράγωνα;

Λύση:

Έστω ότι εφαρμόζουμε την κίνηση σε n οριζόντιες και m κάθετες

γραμμές(n,m 50 ).Τότε το πλήθος των λευκών τετραγώνων που έχουν προκύψει

από τις κινήσεις είναι 50n 50m μείον το πλήθος των μαύρων τετραγώνων που

έχουν προκύψει από τις κινήσεις. Το πλήθος αυτό ισούται με το εμβαδό του

ορθογωνίου που δημιουργείται από τις τομές των γραμμών. Αυτό είναι ίσο με 2nm

σε περίπτωση που οι γραμμές δεν εφάπτονται αυτό δεν αλλάζει το αποτέλεσμα διότι

απλά μεταφέρουμε την γραμμή όσα τετράγωνα χρειάζονται έτσι ώστε οι γραμμές να

εφάπτονται. Έτσι το πλήθος των λευκών τετραγώνων που έχουν προκύψει από τις

κινήσεις είναι ίσο με 50n 50m 2nm 2(25m 25n nm) (1).

α) Για να έχει η σκακιέρα 2011 λευκά τετραγωνάκια αρκεί η εξίσωση

2(25m 25n nm) 2011 να έχει θετικές ακέραιες λύσεις μικρότερες ή ίσες του

50 . Αυτή είναι αδύνατη αφού (2011,2) 1 .Συνεπώς δεν γίνεται να έχουμε 2011

λευκά τετραγωνάκια.

β) Όπως και πριν για να έχει η σκακιέρα 2010 λευκά τετράγωνα αρκεί η εξίσωση

2(25m 25n nm) 2010 να έχει θετικές ακέραιες λύσεις μικρότερες ή ίσες του

50 . Η εξίσωση γράφεται (25 n)(25 m) 380 που έχει λύσεις, πχ (m,n) (40,1) .

Συνεπώς μπορούμε να έχουμε 2010 λευκά τετραγωνάκια στη σκακιέρα.

Page 14: ΠΡΟΤΕΙΝΟΜΕΝΕΣ ΑΣΚΗΣΕΙΣ ΓΙΑ ΜΑΘΗΤΙΚΟΥΣ ΔΙΑΓΩΝΙΣΜΟΥΣ 201 - 300

http://www.mathematica.gr/forum/viewtopic.php?f=109&t=15584

Επιμέλεια: xr.tsif Σελίδα 14

Β τρόπος στο πρώτο ερώτημα.

Έστω σε ένα στάδιο έχουμε M μαύρα και A άσπρα τετράγωνα. Έστω ότι στην

επόμενη κίνηση (αλλαγή χρωματισμού μιας γραμμής ή στήλης) περιέχονται n μαύρα

και (φυσικά) 50 n άσπρα τετράγωνα. Τότε τα μαύρα θα γίνουν

M' M n (50 n) (δηλαδή τα n που χάσαμε συν τα 50 n που κερδίσαμε) και

τα άσπρα θα γίνουν A΄ A (50 n) n .

Εξετάζουμε τώρα την διαφορά M΄ Α΄ . Είναι M΄ Α΄ ... Μ Α 100 4n .

Παρατηρούμε ότι ο αριθμός 100 4n είναι πολλαπλάσιο του 4 . Συμπέρασμα, σε

κάθε βήμα ο αριθμός M A αλλάζει κατά πολλαπλάσιο του 4 . Στην αρχή είναι

M A 2500 0 πολλαπλάσιο του 4 , ενώ αν γίνουν 2011 τα άσπρα (και 489 τα

μαύρα), τότε M A 489 2011 1522 που δεν είναι πολλαπλάσιο του 4 .

Συνεπώς δεν θα φτάσουμε ποτέ σε 2011 άσπρα τετράγωνα.

ΘΕΜΑ 213 (Socrates)

Έστω A ένα υποσύνολο του συνόλου {1,2,...,2009} με 1005 στοιχεία, τέτοιο ώστε

το άθροισμα δύο οποιονδήποτε στοιχείων του να μην ισούται με 2009 , ούτε με

2010 . Πόσα τέτοια σύνολα A υπάρχουν;

Λύση:

Χωρίζουμε το σύνολο {1,2,...,2009} σε δύο υποσύνολα B και Γ όπου το B

περιέχει τα στοιχεία {1,2,...,1004} και το Γ περιέχει τα στοιχεία

{1005,1006,...,2009} . Είναι προφανές ότι το σύνολο Γ είναι ένα από τα ζητούμενα

σύνολα.

Θα αποδείξουμε ότι είναι το μοναδικό. Παρατηρούμε ότι αν μετακινήσουμε ένα

στοιχείο του Γ ( π.χ. το 1005 ) στο σύνολο B τότε πρέπει να αφαιρέσουμε ένα άλλο

στοιχείο (το1004 ) και έτσι το σύνολο θα έχει πάλι 1004 στοιχεία . Έτσι είναι

αδύνατο το B να αποκτήσει 1005 στοιχεία. Αν μετακινήσουμε ένα στοιχείο του B

στο Γ (π.χ. το 1004 ) πρέπει να αφαιρέσουμε δύο άλλα στοιχεία (το 1005 και το

1006 ) και έτσι και αυτό το σύνολο είναι αδύνατο να ξανααποκτήσει 1005 στοιχεία.

Page 15: ΠΡΟΤΕΙΝΟΜΕΝΕΣ ΑΣΚΗΣΕΙΣ ΓΙΑ ΜΑΘΗΤΙΚΟΥΣ ΔΙΑΓΩΝΙΣΜΟΥΣ 201 - 300

http://www.mathematica.gr/forum/viewtopic.php?f=109&t=15584

Επιμέλεια: xr.tsif Σελίδα 15

ΘΕΜΑ 214 (ΣΩΤΗΡΗΣ ΛΟΥΡΙΔΑΣ )

Υπολογίστε τετραψήφιο αριθμό xyzw του δεκαδικού συστήματος που να είναι

τέλειο τετράγωνο όταν επιπλέον ισχύουν οι σχέσεις: xy 3zw 1 και w y 1 .

Λύση:

Η πρώτη δοθείσα σχέση γράφεται: 10x y 30z 3w 1 και αν αντικαταστήσουμε

όπου w y 1 , έχουμε 10x 30z 2y 4 (1).

Παρατηρούμε ότι το πρώτο μέλος τελειώνει σε 0 , άρα και το άθροισμα στο δεύτερο

μέλος πρέπει να τελειώνει σε 0 , αυτό γίνεται μόνο για y 3 ή y 8 . Οπότε έχουμε

δύο περιπτώσεις:

1η περίπτωση: y 3 άρα και w 4 .

η (1) γράφεται 10x 30z 10 δηλαδή x 3z 1 .

για z 0 έχουμε x 1 ,

για z 1 έχουμε x 4 ,

για z 2 έχουμε x 7 ,

δηλαδή τους αριθμούς:1304 , 4314 , 7324 που κανένας δεν είναι τέλειο τετράγωνο

οι υπόλοιπες τιμές για το z απορρίπτονται, γιατί δεν θα υπάρχει αντίστοιχο x .

2η περίπτωση: y 8 άρα w 9 .

η (1) γράφεται x 3z 2

παίρνουμε πάλι περιπτώσεις για το z που μπορεί να πάρει τις τιμές z 0 , z 1 ,

z 2 και βρίσκουμε τους αριθμούς: 2809 , 5819 , 8829 από τους οποίου μόνο ο

αριθμός 2809 είναι τέλειο τετράγωνο ,άρα είναι ο ζητούμενος.

Page 16: ΠΡΟΤΕΙΝΟΜΕΝΕΣ ΑΣΚΗΣΕΙΣ ΓΙΑ ΜΑΘΗΤΙΚΟΥΣ ΔΙΑΓΩΝΙΣΜΟΥΣ 201 - 300

http://www.mathematica.gr/forum/viewtopic.php?f=109&t=15584

Επιμέλεια: xr.tsif Σελίδα 16

ΘΕΜΑ 215 (ΣΩΤΗΡΗΣ ΛΟΥΡΙΔΑΣ )

Να αποδειχθεί ότι η παράσταση:

2 2 2

1 1 1

b c c a a b

, είναι τέλειο

τετράγωνο.

Λύση:

Θέτουμε : x a b , y b c , w c a , oπότε x y w 0 .

Επίσης από την ταυτότητα 2 2 2 2(x y w) x y w 2xy 2yw 2xw έχουμε

2 2 2x y w 2xy 2yw 2xw (1) αφού x y w 0 .

υψώνουμε την (1) στο τετράγωνο οπότε 2 2 2 2 2 2 2

4x y 4y w 4x w 8xyw(x y w)A (2) , όπου A το πρώτο μέλος της

(1). Αφού x y w 0 η (2) γίνεται 2A 4B (3) , όπου 2 2 2 2 2 2

B x y y w x w .

Διαιρούμε τώρα την (3) με 2 2 24x y w και προκύπτει το ζητούμενο,

γιατί 2 2 2B / x y w ισούται με τη δοθείσα παράσταση.

ΘΕΜΑ 216 (ΣΩΤΗΡΗΣ ΛΟΥΡΙΔΑΣ )

Σε τρίγωνο ABC δίνονται: AB 5 , AC 7 και το ύψος AD 4 . Υπολογίστε την

BC , το εμβαδόν του και την ακτίνα ρ του εγγεγραμμένου του κύκλου.

Λύση:

Εύκολα με Πυθαγόρειο βρίσκουμε ότι BC 3 33 , άρα

4(3 33)E 6 2 33

2

.

Page 17: ΠΡΟΤΕΙΝΟΜΕΝΕΣ ΑΣΚΗΣΕΙΣ ΓΙΑ ΜΑΘΗΤΙΚΟΥΣ ΔΙΑΓΩΝΙΣΜΟΥΣ 201 - 300

http://www.mathematica.gr/forum/viewtopic.php?f=109&t=15584

Επιμέλεια: xr.tsif Σελίδα 17

Γνωρίζουμε επίσης ότι 15 33

E τρ 6 2 33 ρ2

, από όπου παίρνουμε και

το ρ , ( τ ημιπερίμετρος και ρ ακτίνα εγγεγραμμένου κύκλου).

ΘΕΜΑ 217 (ΣΩΤΗΡΗΣ ΛΟΥΡΙΔΑΣ )

Δίνεται τρίγωνο ABC και σημείο

Ρ στο εσωτερικό του. Θεωρούμε

D το σημείο τομής της CP με την

AB .( Συμβολικά D CP AB και

E BP AC (σχήμα 1))

1) Αιτιολογείστε γιατί οι

περιγεγραμμένοι κύκλοι στα

τρίγωνα PDB , PCE δεν είναι

δυνατόν να εφάπτονται .

2) Αν οι κύκλοι αυτοί είναι ίσοι

τότε αποδείξτε ότι ισχύει η σχέση

AE AD AC AB και αντίστροφα.

Υπόδειξη για το (1) (σχήμα 2) Η

εγγεγραμμένη γωνία KML

είναι ίση με

την γωνία LK P

(γιατί;)

Page 18: ΠΡΟΤΕΙΝΟΜΕΝΕΣ ΑΣΚΗΣΕΙΣ ΓΙΑ ΜΑΘΗΤΙΚΟΥΣ ΔΙΑΓΩΝΙΣΜΟΥΣ 201 - 300

http://www.mathematica.gr/forum/viewtopic.php?f=109&t=15584

Επιμέλεια: xr.tsif Σελίδα 18

(*) Μεθοδολογία: Όταν έχεις τεμνόμενους κύκλους θεώρησε την κοινή τους χορδή,

μπορεί να χρειαστεί. Επίσης όταν έχεις εφαπτόμενους κύκλους θεώρησε την κοινή

τους εφαπτομένη στο σημείο επαφής τους, μπορεί να χρειαστεί.

Λύση:

1) Έστω ότι οι κύκλοι εφάπτονται. Επίσης θέτουμε CEP x

και BDP y

.Τότε η

γωνία BPC x y

, όμως αυτό είναι άτοπο, γιατί το άθροισμα των γωνιών x και

ECP

δεν μπορεί να ισούται με x y , αφού ECP

δεν μπορεί να έχει μέτρο y . Αυτό

γιατί ECP A y

.

2) Αφού οι κύκλοι είναι ίσοι από το νόμο των ημιτόνων και τις ίσες γωνίες DPB

και

EPC

έχουμε ότι DB EC .

Έτσι: AC AB AE EC AD DB AE AD .

Β τρόπος

1) Στην περίπτωση που οι κύκλοι

αυτοί εφάπτονταν, αυτό θα συνέβαινε

στο σημείο P , αφού ανήκει και στους

δύο κύκλους. Έτσι για αυτούς θα

είχαμε την κάτω σχηματική απόδοση.

Με βάση τώρα και την θεωρία που

ήδη αναφέραμε παίρνουμε:

DBP DPK LPC PEC DB EC, με A DB και A EC,

που είναι άτοπο.

Επομένως οι κύκλοι αυτοί θα έχουν και άλλο ένα κοινό σημείο διαφορετικό του P .

Page 19: ΠΡΟΤΕΙΝΟΜΕΝΕΣ ΑΣΚΗΣΕΙΣ ΓΙΑ ΜΑΘΗΤΙΚΟΥΣ ΔΙΑΓΩΝΙΣΜΟΥΣ 201 - 300

http://www.mathematica.gr/forum/viewtopic.php?f=109&t=15584

Επιμέλεια: xr.tsif Σελίδα 19

2) Στην περίπτωση της ισότητας των κύκλων αυτών θα είχαμε:

DPB CPE τoξ.DB τoξ.CE DB CE.

Από εδώ προκύπτει άμεσα το ζητούμενο.

ΘΕΜΑ 218 (Socrates)

Βρείτε όλους τους διψήφιους αριθμούς AB τέτοιους ώστε AB / A0B .

Λύση:

Απάντηση στο ερώτημα: Αν δεν ξέχασα άλλες τόσες περιπτώσεις γιατί δεν είμαι

καλός στα Λογιστικά, έχουμε τις λύσεις

10 / 100, 15 / 105, 18 / 108, 20 / 200, 30 / 300, 40 / 400, 45 / 405,

50 / 500, 60 / 600, 70 / 700, 80 / 800, 90 / 900 . Εξ υποθέσεως 10A B / 100A B

άρα 10A B / 90A . Τρέχουμε τώρα τον A 1,2,3,4,5,6,7,8,9 .

Π.χ. για τον A 1 ψάχνουμε τους αριθμούς της μορφής 1B / 90 . Αναλογιζόμενοι ότι

οι διαιρέτες του 90 είναι 1,2,3,5,6,9,10,15,18,30,45,90 (δεν θα χρειαστούν όλοι

Page 20: ΠΡΟΤΕΙΝΟΜΕΝΕΣ ΑΣΚΗΣΕΙΣ ΓΙΑ ΜΑΘΗΤΙΚΟΥΣ ΔΙΑΓΩΝΙΣΜΟΥΣ 201 - 300

http://www.mathematica.gr/forum/viewtopic.php?f=109&t=15584

Επιμέλεια: xr.tsif Σελίδα 20

εδώ, αλλά τους καταγράφω γιατί χρειάζονται στις επόμενες περιπτώσεις), βρίσκουμε

τις λύσεις B 0 ή B 5 ή Β 8 .

Και λοιπά, και λοιπά και λοιπά.

Β τρόπος

Για b 0 το ζητούμενο ισχύει.

Για b 0 γράφουμε 100a b k(10a b) 10a(10 k) (k 1)b.

Είναι 10a 10 b οπότε k 1 10 k 10 k 6.

-Αν k 6 έχουμε b 8a οπότε a 1, b 8

-Αν k 7 έχουμε b 5a οπότε a 1, b 5

-Αν k 8 έχουμε 7b 20a αδύνατη.

-Αν k 9 έχουμε 4b 5a οπότε a 4, b 5 .

ΘΕΜΑ 219 (Socrates)

Αν οι θετικοί αριθμοί x,y,z είναι τέτοιοι ώστε

2 2 2

1 1 1 1

x 1 y 1 z 1 2

να δείξετε ότι

3 3 3

1 1 1 1.

x 2 y 2 z 2 3

Λύση:

Αρκεί να δειχθεί ότι 3 2

3 21

x 2 x 1

.

Όμως 3 2 3 3 2

3 2

3 22x 4 3x 3 x x 1 3x

x 2 x 1

,

που ισχύει από ΑΜ – ΓΜ. Το ζητούμενο έπεται.

Page 21: ΠΡΟΤΕΙΝΟΜΕΝΕΣ ΑΣΚΗΣΕΙΣ ΓΙΑ ΜΑΘΗΤΙΚΟΥΣ ΔΙΑΓΩΝΙΣΜΟΥΣ 201 - 300

http://www.mathematica.gr/forum/viewtopic.php?f=109&t=15584

Επιμέλεια: xr.tsif Σελίδα 21

Για να ισχύει η ισότητα πρέπει x 1 . Ομοίως προκύπτει ότι για να ισχύουν οι

επιμέρους ισότητες πρέπει y 1, z 1 . Όμως η δοθείσα σχέση δεν επαληθεύεται για

x y z 1 και συνεπώς η ισότητα δεν μπορεί να ισχύει.

ΘΕΜΑ 220 (Socrates)

Αν οι πραγματικοί αριθμοί a,b,c είναι τέτοιοι ώστε οι αριθμοί ab,bc,ca να είναι

ρητοί, να δείξετε ότι υπάρχουν ακέραιοι x,y,z όχι όλοι μηδέν, τέτοιοι ώστε

ax by cz 0 .

Λύση:

Αν δύο ή τρεις από τους a,b,c είναι 0 , απλό (αν π.χ. a b 0 c παίρνουμε

x y 1,z 0 .

Χωρίς βλάβη της γενικότητας υποθέτουμε ότι ab,bc,ca δεν είναι όλοι 0 . Παίρνουμε

x 2Mbc , y Mac , z Mab όπου M οποιοδήποτε πολλαπλάσιο των

παρονομαστών των ab,bc,ca .

Ελέγχουμε ότι ισχύουν τα ζητούμενα.

ΘΕΜΑ 221 (Socrates)

Σε ένα διαγωνισμό συμμετέχουν 90 μαθητές. Κάθε μαθητής γνώρισε τουλάχιστον

60 άλλους συμμετέχοντες μαθητές. Να αποδείξετε ότι τέσσερις τουλάχιστον μαθητές

έχουν τον ίδιο αριθμό γνωριμιών.

(Η γνωριμία είναι συμμετρική. Αν ο A γνωρίζει τον B , τότε και ο B γνωρίζει τον A .)

Λύση:

Για κάθε 60 n 89 ας γράψουμε n

A για το σύνολο των μαθητών που γνώρισαν

ακριβώς n άλλους μαθητές. Στόχος μας είναι να δείξουμε ότι κάποιο από τα n

A

περιέχει τουλάχιστον τέσσερις μαθητές. Ας υποθέσουμε πως αυτό δεν ισχύει. Άρα

Page 22: ΠΡΟΤΕΙΝΟΜΕΝΕΣ ΑΣΚΗΣΕΙΣ ΓΙΑ ΜΑΘΗΤΙΚΟΥΣ ΔΙΑΓΩΝΙΣΜΟΥΣ 201 - 300

http://www.mathematica.gr/forum/viewtopic.php?f=109&t=15584

Επιμέλεια: xr.tsif Σελίδα 22

nA 3 για κάθε 60 n 89 . Επειδή όμως συνολικά έχουμε 90 μαθητές, τότε

60 8990 | A | | A | 3 3 90 και άρα, για να ισχύει η ισότητα πρέπει

nA 3 για κάθε 60 n 89 . Δηλαδή ακριβώς 3 μαθητές γνώρισαν άλλους 60 ,

ακριβώς 3 γνώρισαν άλλους 61 , ... , ακριβώς 3 γνώρισαν άλλους 89 .

Ας μετρήσουμε τώρα όλες τις γνωριμίες. Έχουμε συνολικά 3(60 61 89)

2

γνωριμίες αφού στο άθροισμα 3(60 61 89) μετράμε κάθε γνωριμία διπλά,

από μία για κάθε ένα από τα δυο άτομα που γνωρίστηκαν μεταξύ τους. Όμως

3(60 61 89) 3 149 30

2 4

ο οποίος δεν είναι ακέραιος. Αυτό είναι αδύνατον

και άρα το ζητούμενο αποδείχθηκε.

Θα μπορούσαμε να γράψουμε την πιο πάνω λύση χρησιμοποιώντας την ορολογία της

θεωρίας γραφημάτων. Δείτε π.χ.

«Ο Γιώργος ισχυρίζεται ότι χθες βρισκόταν σε μια συνάντηση με άλλα εννιά άτομα οι

οποίοι (συμπεριλαμβανομένου του Γιώργου) έκαναν 3,4,4,4,5,5,6,6,7,7 χειραψίες

αντίστοιχα. Ο Ανδρέας είναι σίγουρος πως ο Γιώργος έχει κάνει λάθος στο μέτρημα.

Πως το ξέρει;»

Λύση

Οι χειραψίες γίνονται σε "ζευγάρια",(αν ο x κάνει χειραψία με τον y τότε αυτόματα

έχει κάνει και ο y με τον ) x , άρα συνολικά ζητάμε άρτιο αριθμό χειραψιών, κάτι

που δεν ισχύει στην δική μας περίπτωση.

Ωραία.

Να αναφέρω σχετικά με τον τίτλο ότι μπορούμε να παραστήσουμε αυτήν την

κατάσταση με ένα "γράφημα" το οποίο αποτελείται από 10 σημεία τα οποία

ονομάζουμε κορυφές (μια κορυφή για κάθε άτομο) και στο οποίο ενώνουμε δυο

κορυφές μεταξύ τους με μια γραμμή την οποία ονομάζουμε ακμή αν και μόνο αν τα

αντίστοιχα άτομα έχουν κάνει χειραψία μεταξύ τους.

Page 23: ΠΡΟΤΕΙΝΟΜΕΝΕΣ ΑΣΚΗΣΕΙΣ ΓΙΑ ΜΑΘΗΤΙΚΟΥΣ ΔΙΑΓΩΝΙΣΜΟΥΣ 201 - 300

http://www.mathematica.gr/forum/viewtopic.php?f=109&t=15584

Επιμέλεια: xr.tsif Σελίδα 23

Για κάθε κορυφή v ενός γραφήματος G συμβολίζουμε με d(v) τον αριθμό των

ακμών που περνάνε από αυτήν την κορυφή. (Στην περίπτωσή μας αυτό ισούται με

τον αριθμό των χειραψιών που έκανε το αντίστοιχο άτομο.)

Αυτό που έχει αποδείξει ο Μάριος είναι ότι για κάθε γράφημα G αν αθροίσουμε όλα

τα d(v)θα πάρουμε ένα άρτιο αριθμό. Πιο συγκεκριμένα αυτός ο αριθμός θα ισούται

με 2e(G) όπου με e(G) συμβολίζουμε τον αριθμό των ακμών του γραφήματος.

ΘΕΜΑ 222 (Socrates)

Ένας επταψήφιος αριθμός αποτελείται από διαφορετικά μεταξύ τους ψηφία και είναι

πολλαπλάσιο καθενός από αυτά(τα ψηφία). Να βρεθεί από ποια ψηφία αποτελείται ο

αριθμός.

Λύση:

Ο επταψήφιος 3548160 είναι πολλαπλάσιο όχι μόνο των ψηφίων του αλλά όλων των

0,1,2,...,9 .

Για την κατασκευή του δεν πάμε τελείως στα τυφλά: Αν κρατήσουμε τα ψηφία

1,3,4,5,6,8,0 έχουν άθροισμα 27 οπότε εξασφαλίζουμε ότι φτιάχνουν πολλαπλάσιο

του 3 και 9 . Αν βάλω το 160 στο τέλος του, εξασφαλίζω πολλαπλάσιο των 2,4,5,8 .

Αυτόματα έχω του 6 2 3 και φυσικά των 1και 0 . Μένει ο 7 . Ανακατεύοντας τα

ψηφία πλην των τριών τελευταίων όλο και κάποιο πολλαπλάσιο του 7 θα βρίσκω.

Π.χ. δεν μου κάνει ο 5348160 , αλλά δεν στεναχωριέμαι: δοκιμάζω εναλλάσσοντας

τα 3,5 και, ω του θαύματος, βρήκα πολλαπλάσιο του 7 (αν ξέρω και κριτήριο

διαιρετότητας με το 7 , τόσο το καλύτερο.

ΛΥΣΗ

Το 0 απαγορεύεται να επιλεγεί ως ψηφίο. Επειδή τουλάχιστον ένα από τα 2,4,6,8

θα πρέπει να επιλεγεί (αλλιώς θα είχαμε πενταψήφιο αριθμό) ο αριθμό πρέπει να

είναι πολλαπλάσιο του δύο. Αν ήταν πολλαπλάσιο του πέντε, τότε θα ήταν

πολλαπλάσιο του δέκα και θα έληγε σε 0 , άτοπο. Άρα ούτε το 5 επιτρέπεται να

επιλεγεί. Άρα ο αριθμός περιέχει εφτά ψηφία από τα 1,2,3,4,6,7,8,9 . Αν δεν

Page 24: ΠΡΟΤΕΙΝΟΜΕΝΕΣ ΑΣΚΗΣΕΙΣ ΓΙΑ ΜΑΘΗΤΙΚΟΥΣ ΔΙΑΓΩΝΙΣΜΟΥΣ 201 - 300

http://www.mathematica.gr/forum/viewtopic.php?f=109&t=15584

Επιμέλεια: xr.tsif Σελίδα 24

περιέχει το 9 , τότε το άθροισμα των ψηφίων του αριθμού θα ισούται με

1 2 3 4 6 7 8 31 το οποίο δεν είναι πολλαπλάσιο του 3 . Άρα ούτε και ο

αριθμός θα είναι πολλαπλάσιο του 3 , άτοπο αφού περιέχει το 3 . Επομένως ο

αριθμός περιέχει το ψηφίο 9 και άρα το άθροισμα των ψηφίων του είναι

πολλαπλάσιο του 9 . Επομένως το άθροισμα των ψηφίων πρέπει να ισούται με 36

(το 27 είναι πολύ μικρό και το 45 πολύ μεγάλο) και άρα το άλλο ψηφίο που λείπει

είναι το 4 . Επομένως ο αριθμός περιέχει τα ψηφία 1,2,3,6,7,8,9 .

Η άσκηση δεν ζητάει να βρούμε τον αριθμό αλλά μόνο τα ψηφία οπότε επιτρέπεται

να σταματήσουμε εδώ.

ΘΕΜΑ 223 (Socrates)

Στο επίπεδο δίνονται 51 σημεία με ακέραιες συντεταγμένες και τέτοια ώστε η

απόσταση μεταξύ δύο οποιονδήποτε από αυτά να είναι φυσικός αριθμός. Να δείξετε

ότι τουλάχιστον το 49% αυτών των αποστάσεων είναι άρτιοι αριθμοί.

Λύση:

Χωρίζουμε τα σημεία σε δύο ομάδες.

Α ομάδα: Όσα σημεία έχουν και τις δύο συντεταγμένες άρτιες ή και τις δύο περιττές.

Β ομάδα: Όσα σημεία έχουν μια συντεταγμένη άρτια και μια περιττή.

Παρατηρούμε ότι αν πάρουμε δυο σημεία της A ομάδας, το τετράγωνο της

απόστασής τους είναι άρτιος. Άρα και η απόστασή τους (αφού είναι ακέραιος) είναι

και αυτή άρτιος αριθμός. Το ίδιο συμβαίνει και για κάθε δυο σημεία της B ομάδας.

Αν γράψουμε x για τον αριθμό των σημείων της A ομάδας και y 51 x για τον

αριθμό των σημείων της B ομάδας, τότε έχουμε το πολύ 2

y4

(x y)x

αποστάσεις οι

Page 25: ΠΡΟΤΕΙΝΟΜΕΝΕΣ ΑΣΚΗΣΕΙΣ ΓΙΑ ΜΑΘΗΤΙΚΟΥΣ ΔΙΑΓΩΝΙΣΜΟΥΣ 201 - 300

http://www.mathematica.gr/forum/viewtopic.php?f=109&t=15584

Επιμέλεια: xr.tsif Σελίδα 25

οποίες είναι περιττές. Αυτό δίνει ποσοστό 2

51 1 51

514 100

2

περιττών αποστάσεων.

Άρα τουλάχιστον το 49% των αποστάσεων είναι άρτιες.

ΘΕΜΑ 224 (ΣΩΤΗΡΗΣ ΛΟΥΡΙΔΑΣ )

Έστω τρίγωνο ABC με n 1 n 2 n 1 n n 1 n n 1AB 3·2 3,BC 2 2 2 ,AC 2 2 2 .

1) Προσδιορίστε τις γωνίες του τριγώνου,

2) Υπολογίστε τοn στην περίπτωση που η περίμετρος του τριγώνου είναι:

24 3( 3 1) .

Λύση:

1) Ας θέσουμε για ευκολία nx 2 Τότε, είναι

n 2 na 2 (2 2 1) 3·2 3x, n n1 3 3

b 2 2 1 2 x,2 2 2

( ) n3 3 3 3

c 2 x.2 2

Παρατηρούμε, ότι 2 2 2 2 2 29 27b c x x 9x a ,

4 4 άρα το τρίγωνο είναι ορθογώνιο

με 0

A 90 .

Επίσης είναι a

b2

, άρα 0

B 30 .

οπότε 0

C 60 .

2) Για την περίμετρο του τριγώνου έχουμε

3 3 3 9 3 3 3 3(1 3)a b c 3x x x x x.

2 2 2 2

Άρα πρέπει x 16 δηλαδή n

2 16 οπότε x 4 .

Άλλες λύσεις εδώ :http://www.mathematica.gr/forum/viewtopic.php?f=69&t=12119

Page 26: ΠΡΟΤΕΙΝΟΜΕΝΕΣ ΑΣΚΗΣΕΙΣ ΓΙΑ ΜΑΘΗΤΙΚΟΥΣ ΔΙΑΓΩΝΙΣΜΟΥΣ 201 - 300

http://www.mathematica.gr/forum/viewtopic.php?f=109&t=15584

Επιμέλεια: xr.tsif Σελίδα 26

ΘΕΜΑ 225 (ΣΩΤΗΡΗΣ ΛΟΥΡΙΔΑΣ )

Δίνεται κύκλος (O,R) . Να βρεθεί τρόπος να χωριστεί η επιφάνεια του σε τρία

ισοδύναμα μέρη (δηλαδή μέρη του ίδιου εμβαδού) που για το κάθε ένα από αυτά τα

μέρη να ισχύει: υπάρχει το πολύ ένα τμήμα της γραμμής, από την οποία περιορίζεται,

που να ορίζει ημικύκλιο με την διάμετρο του, εντός του μέρους.

(*) Απλά αναφέρουμε ότι το πρόβλημα γενικεύεται και για χωρισμό σε n ισοδύναμα

μέρη με βάση την ίδια δέσμευση.

Λύση:

Δεν είμαι τόσο βέβαιος ότι καταλαβαίνω τι ακριβώς θα πει «... με την διάμετρο του

εντός του μέρους », αλλά κάνω μία προσπάθεια: Η διάμετρος του μεγάλου κύκλου

έχει χωριστεί σε τρία ίσα μέρη. Εύκολα βλέπουμε ότι τα χρωματισμένα εμβαδά είναι

ίσα μεταξύ τους.

Όμως έβαλα την προϋπόθεση με την

διάμετρο (...που για το κάθε ένα από

αυτά τα μέρη να ισχύει: υπάρχει το πολύ

ένα τμήμα της γραμμής, από την οποία

περιορίζεται, που να ορίζει ημικύκλιο με

την διάμετρο του εντός του μέρους) ,

ώστε να αποκλειστούν άλλες

περιπτώσεις, όπως εκείνη (για

παράδειγμα) του σχήματος που

ακολουθεί και όπου ο μικρός μέσα

κύκλος (με Εμβαδό το ένα τρίτο (1

3) του

Μεγάλου κύκλου), αποτελείται από δύο

τουλάχιστον ημικύκλια μου η διάμετρός

τους να βρίσκεται εντός του (άρα απαγορευμένη αυτή η περίπτωση).

Page 27: ΠΡΟΤΕΙΝΟΜΕΝΕΣ ΑΣΚΗΣΕΙΣ ΓΙΑ ΜΑΘΗΤΙΚΟΥΣ ΔΙΑΓΩΝΙΣΜΟΥΣ 201 - 300

http://www.mathematica.gr/forum/viewtopic.php?f=109&t=15584

Επιμέλεια: xr.tsif Σελίδα 27

Επομένως αν αντικαθιστούσα την αντίστοιχη έκφραση της εκφώνησης από την:

….που για τουλάχιστον ένα από αυτά τα μέρη να υπάρχουν τουλάχιστον δύο

τμήματα της γραμμής από την οποία περιορίζεται που να αποκόπτουν από αυτήν

ημικύκλια με την διάμετρο τους να βρίσκεται εντός του μέρους , τότε θα αποκλείαμε

την περίπτωση «γιν γιάν» που ανέφερες και θα δεχόμασταν εκείνη του σχήματος

που ακολουθεί (κάτω).

Αυτές ήταν οι σκέψεις μου που με οδήγησαν στην συγκεκριμένη εκφώνηση.

(*) Η ακτίνα του μικρού κύκλου είναι: AE R 3

r ,3 3

όπου R είναι η ακτίνα του

μεγάλου κύκλου.

(**) Θα παρακαλούσα τους μικρούς (μόνο σε ηλικία) συναδέλφους να ασχοληθούν

με την απόδειξη της ισότητας των τριών αυτών εμβαδών τόσο στο σχήμα του

Μιχάλη, όσο και σε αυτό που ακολουθεί για την Γεωμετρική τους προπονησούλα.

Page 28: ΠΡΟΤΕΙΝΟΜΕΝΕΣ ΑΣΚΗΣΕΙΣ ΓΙΑ ΜΑΘΗΤΙΚΟΥΣ ΔΙΑΓΩΝΙΣΜΟΥΣ 201 - 300

http://www.mathematica.gr/forum/viewtopic.php?f=109&t=15584

Επιμέλεια: xr.tsif Σελίδα 28

ΘΕΜΑ 226 (ΣΩΤΗΡΗΣ ΛΟΥΡΙΔΑΣ )

Δίνεται τρίγωνο ABC και σημείο P στο εσωτερικό του. Θεωρούμε D το σημείο

τομής της CP με την AB ( Συμβολικά D CP AB και E BP AC . Αποδείξτε

μία ειδική περίπτωση (βασικότατη) του θεωρήματος Miquel :

Αν το τετράπλευρο ADPE είναι εγγράψιμο σε κύκλο τότε το άλλο σημείο τομής T

των δύο κύκλων θα είναι σημείο της BC .

Μπορείτε να διατυπώσετε το αντίστροφο τού παραπάνω προβλήματος και να το

επιλύσετε;

Λύση:

Έστω ότι ο περιγεγραμμένος κύκλος του τριγώνου BDP τέμνει την πλευρά BCστο

σημείο T .

Θα αποδείξουμε ότι το τετράπλευρο TPEC είναι εγγράψιμο σε κύκλο, (είναι η

ισοδύναμη πρόταση της ζητούμενης).Στο αρχικό σχήμα έχουμε:

Αν η γωνία ADP x

, τότε ισχύουν τα εξής σύμφωνα με το γνωστό θεώρημα (για

το οποίο ισχύει και το αντίστροφό του)

Page 29: ΠΡΟΤΕΙΝΟΜΕΝΕΣ ΑΣΚΗΣΕΙΣ ΓΙΑ ΜΑΘΗΤΙΚΟΥΣ ΔΙΑΓΩΝΙΣΜΟΥΣ 201 - 300

http://www.mathematica.gr/forum/viewtopic.php?f=109&t=15584

Επιμέλεια: xr.tsif Σελίδα 29

"Σε κάθε εγγράψιμο τετράπλευρο σε κύκλο, κάθε εξωτερική του γωνία ισούται με

την απέναντι εσωτερική" έχουμε τα εξής:

Η γωνία oBDP 180 x

, η γωνία PEC x

και η γωνία oPTC 180 x

Άρα, το τετράπλευρο TPEC είναι εγγεγραμμένο σε κύκλο, άρα οι δύο κύκλοι για

τους οποίους συζητάμε διέρχονται από σημείο της πλευράς BC .

ΘΕΜΑ 227 (ΔΗΜΗΤΡΗΣ ΙΩΑΝΝΟΥ )

Μπορεί η διαφορά δύο τριψήφιων αριθμών A,B οι οποίοι έχουν τα ίδια ψηφία αλλά

με αντίθετη διάταξη, να είναι τετράγωνο κάποιου φυσικού αριθμού (διάφορου από το

μηδέν);

Λύση:

Έστω ότι υπάρχουν τριψήφιοι αριθμοί A,B οι οποίοι έχουν τα ίδια ψηφία αλλά με

αντίθετη διάταξη . Πρέπει 2abc cba n με a,c {1,2,...,9} και b {0,1,2,...,9} .

2 2 2abc cba n 100a 10b c 100c 10b a n 99a 99c n

2 299(a c) n 9 11(a c) n .

Πρέπει το a c 11 . Άτοπο γιατί τα a,c {1,2,...,9} .

ΘΕΜΑ 228 (Socrates)

Βρείτε όλους τους φυσικούς n για τους οποίους ο αριθμός 1! 4! 7! ... (3n 1)!

είναι τέλειο τετράγωνο ακεραίου.

Λύση:

Οι 1! 1,1! 4! 25 είναι τέλεια τετράγωνα. Θα δούμε ότι δεν υπάρχουν άλλες

περιπτώσεις.

Page 30: ΠΡΟΤΕΙΝΟΜΕΝΕΣ ΑΣΚΗΣΕΙΣ ΓΙΑ ΜΑΘΗΤΙΚΟΥΣ ΔΙΑΓΩΝΙΣΜΟΥΣ 201 - 300

http://www.mathematica.gr/forum/viewtopic.php?f=109&t=15584

Επιμέλεια: xr.tsif Σελίδα 30

Το 1! 4! 7! 5065 δεν είναι τέλειο τετράγωνο. Από κει και πέρα, αφού οι αριθμοί

της μορφής 11!,15!,... λήγουν σε 00 έχουν τουλάχιστον δύο πεντάρια ο καθένας, ως

παράγοντες), τα παραπάνω αθροίσματα είναι της μορφής 100b 65 . Αν ήσαν τέλεια

τετράγωνα, θα ήσαν της μορφής 2100b 65 (10c d) ,0 d 9 .

Παρατηρούμε ότι οι αριθμοί 1! 4! 7! ... είναι "μεγάλοι" (μεγαλύτεροι του 510 )

οπότε c "μεγάλο" (μεγαλύτερο του 50) . Άρα 2 2 2 2 2 2

100c 20cd d 100100b c 1006 0d d 1005 ( c10c d) d (*),

που σημαίνει ότι τα δύο τελευταία ψηφία του είναι ένας από τους 2 2 20 , 1 ,...,9 .

Κανένα όμως από αυτά δεν είναι 65 , οπότε η ισότητα (*) είναι αδύνατη.

ΘΕΜΑ 229 (DEMETRES )

Δίνεται ένα θετικός ακέραιος αριθμός n . Αναδιατάσσουμε τα ψηφία του για να

πάρουμε ένα άλλο ακέραιο αριθμό m . Να εξεταστεί αν το άθροισμά τους μπορεί να

ισούται με 99...9 όπου εμφανίζονται ακριβώς 2011 εννιάρια.

Λύση:

Θα εξετάσουμε περιπτώσεις με λιγότερα εννιάρια και θα δούμε αν μπορεί να βγει

κάποιο συμπέρασμα.

Για δύο εννιάρια, αρκεί τα ψηφία των διψήφιων αριθμών να έχουν άθροισμα 9 .

Για τρία εννιάρια έστω οι αριθμοί abc και η αναδιάταξη του cab .

Τότε 110a 11b 101c 999 Από εδώ συμπεραίνουμε ότι θα πρέπει να είναι

b c 9 . Χρησιμοποιώντας αυτό παίρνουμε 110a 90c 900 11a 9c 90 , που

είναι αδύνατο.

Για τέσσερα εννιάρια, έστω ο αριθμός abcd .

Τότε θα πρέπει τα ψηφία ανά ζευγάρια να έχουν άθροισμα 9 , π.χ. έστω ότι a c 9

και b d 9 . Τότε τοποθετώντας το c ως ψηφίο χιλιάδων, το d ως ψηφίο

εκατοντάδων, το a ως ψηφίο δεκάδων και το bως ψηφίο μονάδων θα έχουμε το

Page 31: ΠΡΟΤΕΙΝΟΜΕΝΕΣ ΑΣΚΗΣΕΙΣ ΓΙΑ ΜΑΘΗΤΙΚΟΥΣ ΔΙΑΓΩΝΙΣΜΟΥΣ 201 - 300

http://www.mathematica.gr/forum/viewtopic.php?f=109&t=15584

Επιμέλεια: xr.tsif Σελίδα 31

απαιτούμενο αποτέλεσμα, αφού

1000(a c) 100(b d) 10(c a) d b 1000·9 100·9 10·9 9 9999

Το συμπέρασμα είναι ότι:

Γενικά οι αριθμοί που θα έχουν άρτιο πλήθος ψηφίων θα έχουν πάντα μια

αναδιάταξη που να δίνει το απαιτούμενο αποτέλεσμα αφού θα μπορούμε πάντα να

τους χωρίζουμε σε ζευγάρια ώστε αυτά να έχουν άθροισμα 9 . Αντίθετα όσοι αριθμοί

θα έχουν περιττό πλήθος ψηφίων δεν θα έχουν αναδιάταξη που να δίνει το

απαιτούμενο αποτέλεσμα αφού δεν θα μπορούμε τα ψηφία τους να τα χωρίσουμε σε

ζευγάρια ώστε να έχουν άθροισμα 9 . Έτσι τελικά δεν γίνεται το άθροισμα ενός

αριθμού και μιας αναδιάταξης των ψηφίων του να είναι ο 99...9 με 2011 εννιάρια

(μήπως χρειάζεται μια πιο ισχυρή απόδειξη..; )

Καλά έκανες και κοίταξες τις μικρές περιπτώσεις και σωστά υποψιάζεσαι ότι δεν

γίνεται να έχουμε περιττό αριθμό εννιαριών. Πρέπει όμως να αποδειχθεί. Προς το

παρόν έχει δώσει μια απόδειξη (*) για τρία εννιάρια αλλά γιατί αυτή η απόδειξη να

γενικεύεται; Πρέπει να το εξηγήσεις.

(*) Και στην απόδειξη με τα τρία εννιάρια υπάρχει ένα πρόβλημα. Έχεις ελέγξει

μόνο μια αναδιάταξη την cab . Υπάρχουν όμως άλλες πέντε αναδιατάξεις που πρέπει

να ελεγχθούν. Οι abc,acb,bac,bca,cba . Για κάθε μία από αυτές πρέπει να δειχθεί

ότι το άθροισμα με το abc δεν μπορεί να ισούται με 999 . Και αν οι περιπτώσεις εδώ

είναι λίγες και μπορούν να ελεγχθούν μία προς μία, για τα 2011 εννιάρια πρέπει να

σκεφτούμε κάτι διαφορετικό. Κάτι που θα δουλεύει για όλες τις περιπτώσεις...

ΠΑΡΑΤΗΡΗΣΕΙΣ

Μια σπουδαία μέθοδος για την απόδειξη μιας συνεπαγωγής είναι η μέθοδος της

αντιθετοαντιστροφής.

Ας πούμε πρώτα τι ονομάζουμε λογική πρόταση.

Page 32: ΠΡΟΤΕΙΝΟΜΕΝΕΣ ΑΣΚΗΣΕΙΣ ΓΙΑ ΜΑΘΗΤΙΚΟΥΣ ΔΙΑΓΩΝΙΣΜΟΥΣ 201 - 300

http://www.mathematica.gr/forum/viewtopic.php?f=109&t=15584

Επιμέλεια: xr.tsif Σελίδα 32

Λογική πρόταση, ονομάζουμε κάθε έκφραση που έχει νόημα και δέχεται έναν και

μόνο από τους χαρακτηρισμούς αληθής (Α) ή ψευδής (Ψ).

ΑΝΤΙΘΕΤΟΑΝΤΙΣΤΡΟΦΗ

Έστω ότι έχουμε δύο λογικές προτάσεις p και q και ας συμβολίσουμε με p,q τις

αρνήσεις των προτάσεων αυτών. Τότε η παρακάτω ισοδυναμία είναι πάντα αληθής:

(p q) (q p) .

Συνεπώς, όταν θέλουμε να αποδείξουμε μια συνεπαγωγή p q , αρκεί να

αποδείχνουμε (αν αυτό είναι ευκολότερο) την συνεπαγωγή:

q p (δηλαδή ότι η άρνηση της qσυνεπάγεται την άρνηση της p .

ΘΕΜΑ 230 (ΔΗΜΗΤΡΗΣ ΙΩΑΝΝΟΥ )

Δίνονται οι ακέραιοι x,y . Αν η παράσταση 2 2x y δεν διαιρείται με το 3 , να

αποδείξετε ότι ακριβώς ένα από τα x,y διαιρείται με το 3 .

Λύση:

Αν υποθέσουμε ότι x 0(mod3) και y 0(mod3) τότε

2 2 2 2x 1(mod3) και y 1(mod3) ,άρα x y 0(mod3) , άτοπο.

ΘΕΜΑ 231 (Socrates)

Να βρεθούν οι φυσικοί αριθμοί n για τους οποίους οι αριθμοί n

n 1 και 2n(2n) 1

είναι πρώτοι.

Λύση:

Θα χρησιμοποιήσουμε το εξής Λήμμα :

"Αν n 1 και a 2 είναι ακέραιοι τέτοιοι ώστε ο n

a 1 είναι πρώτος, τότε (ο a είναι

άρτιος, και) r

n 2 για κάποιο θετικό ακέραιο r ."

Page 33: ΠΡΟΤΕΙΝΟΜΕΝΕΣ ΑΣΚΗΣΕΙΣ ΓΙΑ ΜΑΘΗΤΙΚΟΥΣ ΔΙΑΓΩΝΙΣΜΟΥΣ 201 - 300

http://www.mathematica.gr/forum/viewtopic.php?f=109&t=15584

Επιμέλεια: xr.tsif Σελίδα 33

ΑΠΟΔΕΙΞΗ ΛΗΜΜΑΤΟΣ

Το ότι o a είναι άρτιος είναι προφανές, αφού κάθε πρώτος αριθμός είναι η δύναμη

ενός πολλαπλάσιου του δύο αυξημένη κατά την μονάδα. Δηλαδή εάν p είναι πρώτος,

τότε ισχύει ότι jp (2k) 1 όπου j,k Z .

Αυτό που μένει προς απόδειξη είναι το ότι mj 2 .

Είναι επίσης προφανές αφού εάν είχαμε έναν περιττό αριθμό π.χ. j 3 τότε θα

είχαμε p 9 (για k 1 , ή για άλλη τιμή του k κάποιον άλλο σύνθετο αριθμό) που

δεν είναι πρώτος. Άρα θέλουμε να ισχύει ότι jp (2k) 1 όπου j,k Z και ο j

είναι άρτιος, δηλαδή παίρνει τη μορφή 2m .

ΑΠΟΔΕΙΞΗ

Για n 1 , είναι nn 1 2 και 2n

(2n) 1 5 , άρα η λύση n 1 γίνεται δεκτή.

Βασιζόμενοι στο βοηθητικό πρόβλημα, βλέπουμε ότι αν n 1 , τότε θα είναι rn 2 ,

(με r 0 ) και οι αριθμοί

rn r 2

n 1 2 1 και

r 12n (r 1) 2(2n) 1 2 1

θα είναι πρώτοι.

Από το βοηθητικό πρόβλημα, ξανά θα είναι r k

r 2 2 και r 1 μ(r 1)2 2

, για

κάποιους θετικούς ακέραιους k,μ σαφώς με μ k .

Διαιρώντας κατά μέλη παίρνουμε μ k 1r 1

2r

, κι άρα r 1 που δίνει n 2 .

Εύκολα ελέγχεται ότι το n 2 γίνεται δεκτό, αφού δίνει

nn 1 5 και 2n

(2n) 1 257 , που είναι πρώτοι αριθμοί.

Συνεπώς, οι μόνες λύσεις είναι n 1 και n 2 .

Page 34: ΠΡΟΤΕΙΝΟΜΕΝΕΣ ΑΣΚΗΣΕΙΣ ΓΙΑ ΜΑΘΗΤΙΚΟΥΣ ΔΙΑΓΩΝΙΣΜΟΥΣ 201 - 300

http://www.mathematica.gr/forum/viewtopic.php?f=109&t=15584

Επιμέλεια: xr.tsif Σελίδα 34

ΘΕΜΑ 232 (Socrates)

Αν οι θετικοί ακέραιοι a,b,c,d * είναι τέτοιοι ώστε 2 2ad b bc c , να δείξετε

ότι ο αριθμός 2 2 2 2a b c d είναι σύνθετος.

Λύση:

Είναι

2 2 2 2 2 2 2 2 2 2 2 2a b c d (a d) 2ad b c (a d) 2(b bc c ) b c

2 2(a d) (b c) (a b c d)(a b c d) .

Για να ολοκληρωθεί η απόδειξη χρειάζεται να εξηγήσουμε γιατί καμία από τις

παρενθέσεις δεν ισούται με 1 . Για την πρώτη αυτό είναι φανερό, αφού μιλάμε για

θετικούς ακέραιους. Αν τώρα, υποθέσουμε, ότι η δεύτερη παρένθεση ισούται με 1 ,

θα είναι 2 2 2 2a b c d a b c d .

Καθώς όμως, είναι a,b,c,d * , θα έπρεπε τότε να ισχύει a b c d 1 το

οποίο αντίκειται στην 2 2

ad b bc c .

ΘΕΜΑ 233 (ΣΩΤΗΡΗΣ ΛΟΥΡΙΔΑΣ )

Θεωρούμε ευθεία (ε) και σημεία της Α,B,C ώστε AB BC .Έστω ότι δύο άνισοι

κύκλοι με κέντρα τα σημεία A,C τέμνουν ένα κύκλο με κέντρο το σημείο B στα

σημεία D,E . Να συγκριθούν οι χορδές οι αποκοπτόμενες από τις περιφέρειες με

κέντρα τα σημεία A,B από την ευθεία DE .

Page 35: ΠΡΟΤΕΙΝΟΜΕΝΕΣ ΑΣΚΗΣΕΙΣ ΓΙΑ ΜΑΘΗΤΙΚΟΥΣ ΔΙΑΓΩΝΙΣΜΟΥΣ 201 - 300

http://www.mathematica.gr/forum/viewtopic.php?f=109&t=15584

Επιμέλεια: xr.tsif Σελίδα 35

Λύση:

Οι ΑΜ ,ΒΝ και CG είναι παράλληλες (γιατί;) και αφού AB BC έπεται ότι

MN NG . Αλλά DN NE οπότε MD EG και επομένως ZD EH .

ΘΕΜΑ 234 (ΣΩΤΗΡΗΣ ΛΟΥΡΙΔΑΣ )

Α) Αν a,b και ab 0 , να αποδειχθεί ότι: 2 2 2(a b) a b ,

επίσης αν a,b,c και a b c 0 , να αποδειχθεί ότι: 3 3 3a b c 3abc .

Β) Έστω *m,n

και 0 t 2 . Αποδείξτε ότι:

23 3

3 2

m n 31.

tmnt

Μια υπενθύμιση (Ταυτότητα Euler):

3 3 3 2 2 2a b c 3abc a b c a b c ab bc ca ή

2 2 23 3 3 1

a b c 3abc a b c a b b c c a .2

Page 36: ΠΡΟΤΕΙΝΟΜΕΝΕΣ ΑΣΚΗΣΕΙΣ ΓΙΑ ΜΑΘΗΤΙΚΟΥΣ ΔΙΑΓΩΝΙΣΜΟΥΣ 201 - 300

http://www.mathematica.gr/forum/viewtopic.php?f=109&t=15584

Επιμέλεια: xr.tsif Σελίδα 36

Λύση:

Α) Η πρώτη ισοδυναμεί με την αληθή 2ab 0 . Ας σημειωθεί ότι η ανισότητα είναι

γνήσια αν ab 0 (*).

Η δεύτερη είναι άμεση από την ταυτότητα Euler.

B) Έχουμε 2 2 2 2m n ( mnt) m n 2mn 0 , οπότε από το προηγούμενο

2 3 2 3 3 2 2(m ) (n ) ( mnt) 3(m )(n )( mnt) .

Διαιρώντας με το 3(mnt) έχουμε

6 6

3 2

m n 31

(mnt) t

.

Τελειώνουμε παρατηρώντας (χρήση του Α) και του (*)) ότι 3 3 2 6 6(m n ) m n .

ΘΕΜΑ 235 (ΔΗΜΗΤΡΗΣ ΙΩΑΝΝΟΥ )

Να αποδείξετε ότι ανάμεσα από 79 διαδοχικούς φυσικούς αριθμούς, μπορούμε να

βρούμε έναν, που το άθροισμα των ψηφίων του να είναι διαιρετό με 13 . Ισχύει η

πρόταση για 78 διαδοχικούς φυσικούς;

Λύση:

Εξετάζουμε τον μικρότερο από τους δοθέντες 79 αριθμούς.

Είναι της μορφής A00,A01,...,A99 .

α) Αν ο μικρότερος αυτός αριθμός είναι στο διάστημα A00,A01,...,A20 τότε οι 79

αριθμοί συμπεριλαμβάνουν σίγουρα τους A20,A21,...,A78 και ειδικά

συμπεριλαμβάνουν τους {A20,A21,...,A29} {A75,A76,A77} . Τα υπόλοιπα

μόντουλο 13 των δύο τελευταίων ψηφίων των (δεκατριών) αυτών αριθμών είναι,

αντίστοιχα, {2,3,...,10,11} {12,0,1} . Δηλαδή περιλαμβάνουν όλα τα δυνατά

υπόλοιπα δια 13 , συνεπώς κάποιος από αυτούς είναι πολλαπλάσιο του 13 (ποιος

ακριβώς εξαρτάται από το άθροισμα των ψηφίων του A ).

Page 37: ΠΡΟΤΕΙΝΟΜΕΝΕΣ ΑΣΚΗΣΕΙΣ ΓΙΑ ΜΑΘΗΤΙΚΟΥΣ ΔΙΑΓΩΝΙΣΜΟΥΣ 201 - 300

http://www.mathematica.gr/forum/viewtopic.php?f=109&t=15584

Επιμέλεια: xr.tsif Σελίδα 37

β) Αν ο μικρότερος αυτός αριθμός είναι στο διάστημα A21,...,A60 τότε οι 79

αριθμοί συμπεριλαμβάνουν σίγουρα τους A60,A61,...,A99 . Ειδικά

συμπεριλαμβάνουν τους {A60,A61,...,A66} {A67,A68,A69} {A97,A98,A99} .

Τα υπόλοιπα μόντουλο 13 των δύο τελευταίων ψηφίων των (δεκατριών) αυτών

αριθμών είναι, αντίστοιχα, {6,7,...,12} {0,1,2} {3,4,5} . Δηλαδή, όπως πριν,

περιλαμβάνουν όλα τα δυνατά υπόλοιπα δια 13 και συνεπώς κάποιος από αυτούς

είναι πολλαπλάσιο του 13 .

γ) Αν ο μικρότερος αυτός αριθμός είναι στο διάστημα A61,...,A99 , τότε το πολύ

39 από τους 79 αριθμούς βρίσκονται στο A61,...,A99 . Οι υπόλοιποι, που είναι

τουλάχιστον 40 το πλήθος, συμπεριλαμβάνουν τους B00,B01,...,B39 (ο B00 είναι

ο επόμενος του A99). Ειδικά περιλαμβάνουν τους 13 αριθμούς

{B00,B01,...,B09} {B19,B29,B39} . Τα υπόλοιπα μόντουλο 13 των δύο

τελευταίων ψηφίων των αριθμών αυτών είναι, αντίστοιχα, {0,1,2,...,9} {10,11,12} .

Δηλαδή περιλαμβάνουν όλα τα δυνατά υπόλοιπα δια 13 και συνεπώς κάποιος από

αυτούς είναι πολλαπλάσιο του 13 .

Οι τρεις περιπτώσεις τελειώνουν την απόδειξη.

Δεν έφτιαξα ακόμη παράδειγμα 78 αριθμών που δεν έχουν την παραπάνω ιδιότητα.

Είναι σαφές και προφανές πώς θα κατασκευαστούν με βάση την περίπτωση γ).

Κανείς από τους 78 διαδοχικούς αριθμούς

9999999961,9999999962,...,9999999999,10000000000,...,10000000038 , δεν έχει

άθροισμα ψηφίων ένα πολλαπλάσιο του 13 .

Μπορεί να κάνει κανείς απευθείας έλεγχο: Οι μεν της μορφής 9

9999999961,9999999962,...,9999999999 έχουν άθροισμα ψηφίων ίσα με

{79,80,...,85} {79,80,...,88} {80,81,...,89} {81,82,...,90} και οι

10000000000,...,10000000038 έχουν

{1,2,...,10} {2,3,...,11} {3,4....,12} {4,5,...,12} . Κανένας από αυτούς δεν είναι

πολλαπλάσιο του 13 , και επαληθεύσαμε .

Page 38: ΠΡΟΤΕΙΝΟΜΕΝΕΣ ΑΣΚΗΣΕΙΣ ΓΙΑ ΜΑΘΗΤΙΚΟΥΣ ΔΙΑΓΩΝΙΣΜΟΥΣ 201 - 300

http://www.mathematica.gr/forum/viewtopic.php?f=109&t=15584

Επιμέλεια: xr.tsif Σελίδα 38

Πως σκεφτόμαστε; Με οδηγό πάντα το γ) της απόδειξης , εξετάζουμε τους 78

αριθμούς της μορφής A61,...,A99,B00,B01,...,38 . Το A είναι να το επιλέξουμε.

Το άθροισμα των δύο τελευταίων ψηφίων των αριθμών με " A " έχουν υπόλοιπο

μόντουλο 13 (ελέγχουμε με το χέρι) τους {0,1,...,5,7,...,12} (δηλαδή χάνουν τον 6 ).

Όμοια, οι αριθμοί "B " χάνουν το 12 . Οπότε ψάχνουμε αριθμό A με άθροισμα

s(A) ψηφίων ίσο με 7mod13 (ώστε οι " A " να χάνουν το 0mod13 ) και συγχρόνως

s(A 1) να είναι ίσο με 1mod13 (ώστε και οι "B " να χάνουν το 0mod13 ). Το

προφανές είναι να ψάξουμε αριθμό της μορφής 9999...9 γιατί η δεύτερη συνθήκη

γίνεται τετριμμένη ενώ με δοκιμές επιλέγουμε όσα εννιάρια χρειαζόμαστε για να

ισχύει η πρώτη συνθήκη (πήρα 8 μια και 8 9 7mod13 ).

ΘΕΜΑ 236 (ΔΗΜΗΤΡΗΣ ΙΩΑΝΝΟΥ )

Έστω 1 2 7

a ,a ,...,a δοσμένα ευθύγραμμα τμήματα με 1 2 7

1 a a ... a 10 .

Να αποδείξετε ότι 3 τουλάχιστον από αυτά είναι πλευρές τριγώνου.

Λύση:

Αν δεν υπήρχαν τρία που κατασκευάζουν τρίγωνο, τότε για οποιαδήποτε τρία με

a b c θα ίσχυε a b c .

Τώρα, αφού 2 1

a a 1 θα έχουμε 3 2 1

a a a 1 1 2 , 4 3 2

a a a 2 1 3 ,

5 4 3a a a 3 2 5 ,

6 5 4a a a 5 3 8 και τέλος

7 6 5a a a 8 5 13 ,

άτοπο αφού 7

a 10 .

ΘΕΜΑ 237 (Socrates)

Να δείξετε ότι κάθε τέλειος κύβος γράφεται ως διαφορά τετραγώνων ακεραίων.

Λύση:

Αν ο n είναι περιττός τότε

2 23 3

3 n 1 n 1n

2 2

Page 39: ΠΡΟΤΕΙΝΟΜΕΝΕΣ ΑΣΚΗΣΕΙΣ ΓΙΑ ΜΑΘΗΤΙΚΟΥΣ ΔΙΑΓΩΝΙΣΜΟΥΣ 201 - 300

http://www.mathematica.gr/forum/viewtopic.php?f=109&t=15584

Επιμέλεια: xr.tsif Σελίδα 39

Αν ο n είναι άρτιος τότε

2 23 3

3 n 4 n 4n

4 4

Β τρόπος:

2 22 2

3 n n n nn

2 2

Πως το βρήκαμε; Θέλουμε 3 2 2n x y (x y)(x y) . Την πιο πάνω λύση την

παίρνουμε αν λύσουμε το σύστημα 2x y n και x y n . (Ο Αλέξανδρος λύνει το

σύστημα 3x y n και x y 1 στην περίπτωση που ο n είναι περιττός και το

σύστημα 2

nx y

2 και x y 2 στην περίπτωση που ο n είναι άρτιος. Θέλει μόνο

λίγη προσοχή αφού οι λύσεις που θα βρούμε είναι δεκτές μόνο αν είναι ακέραιες.)

ΘΕΜΑ 238 (Socrates)

Να βρεθεί ο μέγιστος αριθμός διαδοχικών ακεραίων που μπορούν να γραφούν στη

μορφή 2 2n 2m , m,n .

Λύση:

Ανάμεσα σε 6 διαδοχικούς φυσικούς κάποιος είναι της μορφής 8k 5 ή της μορφής

8k 7 . Όμως ο αριθμός 2 2

n 2m δεν είναι ποτέ αυτής της μορφής.

Άρα ο μεγαλύτερος αριθμός διαδοχικών ακεραίων που μπορούν να γραφτούν με

αυτή τη μορφή είναι 5 .

Όμως 2 2

0 0 2·0 , 2 2

1 1 2·0 , 2 2

2 0 2·1 , 2 2

3 1 2·1 , 2 2

4 2 2·0 .

Άρα είναι και ο μέγιστος.

Page 40: ΠΡΟΤΕΙΝΟΜΕΝΕΣ ΑΣΚΗΣΕΙΣ ΓΙΑ ΜΑΘΗΤΙΚΟΥΣ ΔΙΑΓΩΝΙΣΜΟΥΣ 201 - 300

http://www.mathematica.gr/forum/viewtopic.php?f=109&t=15584

Επιμέλεια: xr.tsif Σελίδα 40

ΘΕΜΑ 239 (Socrates)

Να βρείτε όλες τις τριάδες θετικών ακεραίων (a,b,c)τέτοιες ώστε [a,b,c] a b c

όπου [a,b,c] το ελάχιστο κοινό πολλαπλάσιο των a,b,c .

Λύση:

Ας υποθέσουμε ότι c b a .

Προφανώς, c a , αφού διαφορετικά θα ήταν c b a , και [a,b,c] a a b c .

Εξ ορισμού, είναι [a,b,c] xa yb zc για κάποιους θετικούς ακέραιους x,y,z .

Από την υπόθεση, λοιπόν, είναι (z 1)c a b , οπότε a b

0 z 1 2c c

,

κι άρα z 1 1 , δηλαδή z 2 και c a b .

Αν ήταν b a , τότε c 2a και [a,b,c] [a,a,2a] 2a a b c , άτοπο.

Συνεπώς, είναι b a .

Αλλά, από yb 2(a b) έπεται ότι 2a (y 2)b , κι άρα το b , που είναι a

διαιρεί το 2a 2b .

Συνεπώς, θα είναι b 2a , κι άρα c 3a . Προφανώς, τότε ικανοποιείται η

[a,b,c] a b c .

Αποδείξαμε, λοιπόν, ότι αν c b a και [a,b,c] a b c , τότε c 3a και b 2a .

Θεωρώντας όλες τις μεταθέσεις των τριάδων αυτών, συμπεραίνουμε ότι οι

ζητούμενες λύσεις είναι (a,b,c) ~ (t,2t,3t) για κάποιο θετικό ακέραιο t .

ΘΕΜΑ 240 (Atemlos )

Να δείξετε ότι ο 20k 4 10k 2

a a 1 είναι σύνθετος για

*a N,a 1,k N .

Page 41: ΠΡΟΤΕΙΝΟΜΕΝΕΣ ΑΣΚΗΣΕΙΣ ΓΙΑ ΜΑΘΗΤΙΚΟΥΣ ΔΙΑΓΩΝΙΣΜΟΥΣ 201 - 300

http://www.mathematica.gr/forum/viewtopic.php?f=109&t=15584

Επιμέλεια: xr.tsif Σελίδα 41

Λύση:

Άμεση από την 4 2 2 2x x 1 (x x 1)(x x 1) . Εδώ έχουμε 5k 1

x a .

Δηλαδή 20k 4 10k 2 4(5k 1) 2(5k 1) (5k 1) 5 )4 2( k 1a a 1 a a 1 ( )a 1)(a

.

ΘΕΜΑ 241 (ΔΗΜΗΤΡΗΣ ΙΩΑΝΝΟΥ )

Έστω 1

a Z , a R 0a

. Να αποδείξετε ότι n

n

1a Z

a , για κάθε n N .

Λύση:

n 2 n n 1

n 2 n n 1

1 1 1 1a a (a )(a )

a a a a

(1).

Έστω ότι αν ο 1

aa

είναι ακέραιος τότε m

m

1a

a είναι επίσης ακέραιος για κάθε

m n τότε απο την ταυτότητα (1) θα ισχύει και για m n , άρα θα ισχύει για κάθε

φυσικό n .

ΘΕΜΑ 242 (ΔΗΜΗΤΡΗΣ ΙΩΑΝΝΟΥ )

Να βρείτε τα έξι τελευταία ψηφία του αριθμού 1095 , όταν ο αριθμός αυτός γραφτεί

στο δυαδικό σύστημα αρίθμησης.

Λύση:

Αρκεί να βρούμε το 109

5 mod64

Αφού το 36

5 το υπολογίζει ... το κομπιουτεράκι μου = 49mod64 ,

στη συνέχεια μου υπολογίζει το 3

49 5 21mod64 και μετά πατώντας bin

"γλυτώνω" τις διαιρέσεις: 010101 .

Page 42: ΠΡΟΤΕΙΝΟΜΕΝΕΣ ΑΣΚΗΣΕΙΣ ΓΙΑ ΜΑΘΗΤΙΚΟΥΣ ΔΙΑΓΩΝΙΣΜΟΥΣ 201 - 300

http://www.mathematica.gr/forum/viewtopic.php?f=109&t=15584

Επιμέλεια: xr.tsif Σελίδα 42

Β τρόπος

Αν δεν θέλαμε κομπιουτεράκι, θα μπορούσαμε να πούμε: Από Euler είναι φ(64)

5 1mod64 και αφού (από τον τύπο για την φ ) είναι φ(64) 32 , ισχύει

325 1mod64 . Άρα 109 13 32 3 13

5 5 (5 ) 5 mod64 και λοιπά.

Αν δεν θέλαμε καθόλου Euler αλλά να μείνουμε σε "πολύ στοιχειώδη" Μαθηματικά,

θα μπορούσαμε να κατεβάζαμε σταδιακά τον μεγάλο εκθέτη λέγοντας, π.χ.,

109 3 36 36 36 95 5 (5 ) 5 (125) 5 ( 3) 5 81

9 4 2 25 17 5 17 17 5 17 (17 ) 5 17 21mod64 , και λοιπά.

Γ τρόπος

Ας επανέλθω σε αυτήν την άσκηση με ακόμη μια μέθοδο. Χρειάζεται να γνωρίζουμε

το http://en.wikipedia.org/wiki/Binomial_theorem (διωνυμικό θεώρημα).

Όπως και στις προηγούμενες λύσεις, ψάχνουμε το 1095 mod64 . Έχουμε

109 109109 109

5 (1 4) 1 4 16 mod641 2

Όμως 109 3mod16 και 109

2mod42

επομένως

1095 1 12 32 21mod64 .

Η ίδια μέθοδος δίνει επίσης ότι n 25 8n 4n 1mod64 .

Δ τρόπος

Θα χρησιμοποιήσω την ταυτότητα

n n n 1 n 2 2 n 3 3 nn(n 1) n(n 1)(n 2)(x y) x nx y x y x y ... y

1 2 2 3

Έχουμε τώρα: 109 n n 1 6 5

n n 1 6 5 1 05 a 2 a 2 ... a 2 a 2 ... a 2 a

Page 43: ΠΡΟΤΕΙΝΟΜΕΝΕΣ ΑΣΚΗΣΕΙΣ ΓΙΑ ΜΑΘΗΤΙΚΟΥΣ ΔΙΑΓΩΝΙΣΜΟΥΣ 201 - 300

http://www.mathematica.gr/forum/viewtopic.php?f=109&t=15584

Επιμέλεια: xr.tsif Σελίδα 43

όπου τα n n 1 0

a ,a ,...,a

παίρνουν τιμές 0 ή 1 .

Άρα έχουμε:

109 n 6 6 n 7 6 6 5

n n 1 6 5 1 0 05 a 2 2 a 2 2 ... a 2 a 2 ... a 2 a

109 n 6 n 7 6 5 4

n n 1 6 5 4 05 (a 2 a 2 ... a ) 2 a 2 a 2 ... a

Συνεπώς ο αριθμός 5 4

5 4a 2 a 2 ... a είναι το υπόλοιπο της διαίρεσης του 109

5 με

το 62 .

Και άρα τα τελευταία ψηφία του αριθμού που ζητάμε είναι τα 5 4 0

a ,a ,...,a .

Έχουμε τώρα:

109 2 109 109 108 2 107 2 2109 1085 (1 2 ) 1 109 1 (2 ) 1 (2 )

2

106 2 3 2 109 2 4 6109 108 1071 (2 ) ... (2 ) 1 109 2 109 54 2 k 2

2 3

, όπου k

φυσικός αριθμός

Άρα: 109 6 5 3 2 2 6 5 4 65 1 (2 2 2 2 1) 2 (2 45)(2 22) 2 k 2

8 7 5 4 2 15 10 9 4 61 2 2 2 2 2 2 2 22 2 45 45 22 2 k 2

4 2 5 4 61 2 2 2 45 22 2 m 2 , όπου m N .

Άρα 109 2 4 5 5 4 4 65 1 2 2 2 (2 13)(2 6) 2 m 2

2 4 5 13 9 8 4 61 2 2 2 2 6 2 13 2 78 2 m 2

2 4 6 2 5 3 2 4 61 2 79 2 m 2 1 2 (2 2 2 1) 2 m 2

2 9 7 6 4 6 2 4 61 2 2 2 2 2 m 2 1 2 2 t 2 όπου t είναι φυσικός αριθμός.

Άρα το υπόλοιπο που ζητάμε είναι 5 4 3 2 1 0

0 2 1 2 0 2 1 2 0 2 1 2

και τα τελευταία 6 ψηφία που ζητάμε είναι τα 010101 .

Page 44: ΠΡΟΤΕΙΝΟΜΕΝΕΣ ΑΣΚΗΣΕΙΣ ΓΙΑ ΜΑΘΗΤΙΚΟΥΣ ΔΙΑΓΩΝΙΣΜΟΥΣ 201 - 300

http://www.mathematica.gr/forum/viewtopic.php?f=109&t=15584

Επιμέλεια: xr.tsif Σελίδα 44

ΘΕΜΑ 243 (Socrates)

Αν οι θετικοί ακέραιοι x,y,z,t,a,b είναι τέτοιοι ώστε xt yz 1 και x a z

y b t να

δείξετε ότι ab (x z)(y t) .

Λύση:

Έχουμε

1 x z x a a z bx ay at bz 1 1 t y

yt y t y b b t yb bt yb

bt byt

,

οπότε b t y (1).

Ομοίως, είναι

1 t y t b b y ta bz bx ay 1 1 x z

xz z x z a a x az ax az ax axz

,

οπότε a x z (2).

Πολλαπλασιάζοντας τις (1), (2) κατά μέλη παίρνουμε τη ζητούμενη ανισότητα.

ΘΕΜΑ 244 (Socrates)

Αν x και y είναι ακέραιοι, τότε ο αριθμός x y είναι επίσης ακέραιος.

Η πράξη ικανοποιεί τα ακόλουθα:

• x (y z) (x y) z, για όλους τους ακέραιους x,y,z .

• (y z) x (y x) 2z, για όλους τους ακέραιους x,y,z .

• 1 1 1.

Να υπολογίσετε τον αριθμό 25 10 .

Page 45: ΠΡΟΤΕΙΝΟΜΕΝΕΣ ΑΣΚΗΣΕΙΣ ΓΙΑ ΜΑΘΗΤΙΚΟΥΣ ΔΙΑΓΩΝΙΣΜΟΥΣ 201 - 300

http://www.mathematica.gr/forum/viewtopic.php?f=109&t=15584

Επιμέλεια: xr.tsif Σελίδα 45

Bonus: Δείξτε ότι: a b 2a b .

Λύση:

25 10 (24 1) 24 10 2 24 1 9) 2 210 ( 14 1 7 (1 23) 7

1 2 23 7 1 46 7 401 .

Βonus

1 1 1 (a 1) a 1 1 (a 1) 1 2a 1( )

(a 1) 1 2a 1 a 1 2 2a 1 a 1 2a 1

a b (1 b) 2a 1 a b (1 b) 2a 1 a b 2a b( ) .

ΘΕΜΑ 245 (Socrates)

Σε ένα διαγωνισμό τοξοβολίας συμμετέχουν 30 αθλητές. Ο στόχος χωρίζεται σε δύο

ζώνες, τις A και B . Αν το βέλος χτυπήσει στη ζώνη A , ο αθλητής κερδίζει 10

βαθμούς ενώ αν χτυπήσει στη ζώνη B , κερδίζει 5 βαθμούς. Δεν δίνονται βαθμοί για

τα βέλη που δεν βρίσκουν το στόχο. Κάθε αθλητής ρίχνει 16 βέλη. Στο τέλος του

διαγωνισμού, παρατηρήθηκε ότι πάνω από το 50% των βελών χτύπησαν στη ζώνη

B , ενώ ο αριθμός των βελών που χτύπησαν στη ζώνη A είναι ίσος με τον αριθμό

αυτών που δεν βρήκαν το στόχο.

Να δείξετε ότι υπάρχουν δύο αθλητές με την ίδια βαθμολογία.

Λύση:

Ονομάζουμε x,y,z τον αριθμό των βελών που σαν σύνολο θα χτυπήσουν τις ζώνες

A,Β,Γ αντίστοιχα (όπου Γ η εκτός στόχου "ζώνη").

Προφανώς είναι x y z 30 16 480 και επιπλέον ισχύει y 240 και x z με

maxx 119 .

Τώρα καλούμε k k k

x ,y ,z τον αριθμό των βελών που ένας τοξοβόλος k , με

k {1,2,3,...,30} Δ θα ρίξει στις ζώνες A,Β,Γ αντίστοιχα. Αφού κάθε τοξοβόλος

Page 46: ΠΡΟΤΕΙΝΟΜΕΝΕΣ ΑΣΚΗΣΕΙΣ ΓΙΑ ΜΑΘΗΤΙΚΟΥΣ ΔΙΑΓΩΝΙΣΜΟΥΣ 201 - 300

http://www.mathematica.gr/forum/viewtopic.php?f=109&t=15584

Επιμέλεια: xr.tsif Σελίδα 46

ρίχνει 16 βέλη έχουμε k k k

x y z 16 (*) . Δεδομένου του ότι ο καθένας παίρνει

10 πόντους για τη ζώνη A , 5 για τη Β και 0 για τη Γ , το συνολικό σκορ κάθε

τοξοβόλου είναι (*)

k k k k k k k kS 10x 5y 0z 10x 5y 80 5(x z ) .

Εξ αυτού συμπεραίνουμε ότι για να έχουν 2 συμμετέχοντες k,k΄ το ίδιο σκορ είναι:

k k k΄ k΄S(k) S(k΄) x z x z .

Θα υποθέσουμε ότι κανένας τοξοβόλος δεν έχει ίδιο σκορ με κάποιον άλλο και

συνεπώς k k k΄ k΄

S(1) S(2) ... S(30) x z x z για κάθε k,k΄ Δ με k k΄ .

Από περιστεροφωλιά υπάρχουν 15 ομόσημες παραστάσεις k k

x z , έστω θετικές με

διαφορετικό ακέραιο αποτέλεσμα και άρα το ελάχιστο άθροισμά τους είναι:

1 2 ... 15 120 όμως k k k

x x z και άρα το ελάχιστο άθροισμα των 15 αυτών

kx είναι 120 τη στιγμή που

maxx 119 και καταλήγουμε σε αντίφαση ( υπόψη ότι

30

kk 1

x x 120

). Συνεπώς 2 αθλητές θα έχουν σίγουρα ίδιο σκορ.

ΘΕΜΑ 246 (ΔΗΜΗΤΡΗΣ ΙΩΑΝΝΟΥ )

Να λυθεί η εξίσωση: (x 4)(x 5)(x 6)(x 7) 1680 .

Λύση:

Αν τεθεί x 7 y , η εξίσωση γράφεται y(y 1)(y 2)(y 3) 1 1681 , δηλαδή

2 2(y 3y 1) 1681 .

Επομένως αναγόμαστε στη λύση των εξισώσεων

2y 3y 1 41 και 2

y 3y 1 41 .

Προφανώς, η δεύτερη εξίσωση είναι αδύνατη, ενώ η πρώτη έχει τις ρίζες 8 , 5

Άρα η αρχική έχει τις ρίζες 1 , 12 .

Page 47: ΠΡΟΤΕΙΝΟΜΕΝΕΣ ΑΣΚΗΣΕΙΣ ΓΙΑ ΜΑΘΗΤΙΚΟΥΣ ΔΙΑΓΩΝΙΣΜΟΥΣ 201 - 300

http://www.mathematica.gr/forum/viewtopic.php?f=109&t=15584

Επιμέλεια: xr.tsif Σελίδα 47

Φυσικά, για να λύσει κάποιος την αρχική εξίσωση δεν είναι απαραίτητο να θυμάται

την ταυτότητα 2 2x(x 1)(x 2)(x 3) 1 (x 3x 1) .

Είναι καλό όμως, να θυμόμαστε ότι το γινόμενο τεσσάρων διαδοχικών ακεραίων

αυξημένο κατά ένα είναι τέλειο τετράγωνο!

Β τρόπος

Στο πρώτο μέλος της εξίσωσης έχουμε τέσσερις διαδοχικούς αριθμούς, άρα

υποψιαζόμαστε για τέλειο τετράγωνο. Κάνουμε τις πράξεις ως εξής: Επιμεριστική

την πρώτη με την τέταρτη και τις άλλες δύο μαζί.

Θέτουμε: 2 2x 11x 28 a , x 11x 30 a 2 .

Η εξίσωση γίνεται 2a 2a 1680 . Προσθέτουμε 1 και έτσι

2 2 2(x 11x 28 1) 41 . Οπότε αρκεί να λύσουμε τις εξισώσεις:

2x 11x 29 41 και την 2

x 11x 29 41 .

Η πρώτη δίνει λύσεις: x 12 ή x 1 , ενώ η δεύτερη είναι αδύνατη.

ΘΕΜΑ 247 (ΔΗΜΗΤΡΗΣ ΙΩΑΝΝΟΥ )

Χρωματίζουμε όλα τα σημεία του επιπέδου με 2 χρώματα. Να δείξετε ότι υπάρχουν

δύο τουλάχιστον σημεία του επιπέδου που έχουν το ίδιο χρώμα και απέχουν μεταξύ

τους απόσταση 1 .

Λύση:

Εξετάζουμε οποιοδήποτε ισόπλευρο τρίγωνο πλευράς 1 . Δύο κορυφές του

οπωσδήποτε έχουν το ίδιο χρώμα.

Page 48: ΠΡΟΤΕΙΝΟΜΕΝΕΣ ΑΣΚΗΣΕΙΣ ΓΙΑ ΜΑΘΗΤΙΚΟΥΣ ΔΙΑΓΩΝΙΣΜΟΥΣ 201 - 300

http://www.mathematica.gr/forum/viewtopic.php?f=109&t=15584

Επιμέλεια: xr.tsif Σελίδα 48

ΘΕΜΑ 248 (ΔΗΜΗΤΡΗΣ ΙΩΑΝΝΟΥ )

Χρωματίζουμε όλα τα σημεία του επιπέδου με τρία χρώματα. Να αποδειχθεί ότι

υπάρχουν δύο τουλάχιστον σημεία του επιπέδου που έχουν το ίδιο χρώμα και

απέχουν μεταξύ τους απόσταση ίση με 1 .

Λύση:

Θεωρούμε πάνω στο επίπεδο δύο ρόμβους ABCD , AEZH οι οποίοι έχουν κοινή

την κορυφή τους A , πλευρά ίση με 1 και την γωνία της κορυφής A ίση με o60

μοίρες. Επίσης είναι με τέτοιον τρόπο τοποθετημένοι ώστε η απόσταση ZC να είναι

ίση με μονάδα.

Από την αρχή του περιστερώνα, (αφού έχουμε 7 κορυφές και 3 χρώματα) θα

υπάρχουν 3 τουλάχιστον κορυφές που θα έχουν το ίδιο χρώμα. Επίσης από την ίδια

αρχή, δύο τουλάχιστον από αυτά θα ανήκουν στον ίδιο ρόμβο. Εύκολα τώρα

διαπιστώνουμε ότι όπως και να θεωρήσουμε τα τρία σημεία του αυτού χρώματος,

δύο τουλάχιστον, θα απέχουν μεταξύ τους απόσταση 1 .

Β τρόπος

Υπάρχει και εδώ (με επιπλέον ερώτημα).

ΘΕΜΑ 249 (ΔΗΜΗΤΡΗΣ ΙΩΑΝΝΟΥ )

Να αποδείξετε ότι η παράσταση 25

A (n 2 2n 3)2

είναι τέλειο τετράγωνο

ακεραίου, αν γνωρίζουμε ότι η A είναι ακέραιος αριθμός (n N )

Λύση:

Αφού η παράσταση είναι ακέραιος τότε η ρίζα είναι ρητός !

Έστω a

2n 3b

όπου a,b ακέραιοι με bμη μηδενικό τότε 2 2

2

(a 3b )n

2b

.

Page 49: ΠΡΟΤΕΙΝΟΜΕΝΕΣ ΑΣΚΗΣΕΙΣ ΓΙΑ ΜΑΘΗΤΙΚΟΥΣ ΔΙΑΓΩΝΙΣΜΟΥΣ 201 - 300

http://www.mathematica.gr/forum/viewtopic.php?f=109&t=15584

Επιμέλεια: xr.tsif Σελίδα 49

Τώρα αν στην αρχική παράσταση θέσουμε όπου 2 2

2

(a 3b )n

2b

και A m ακέραιο

τότε μετά από πράξεις καταλήγουμε ότι 2 225(a b) m(2b) , άρα m τέλειο

τετράγωνο.

Β τρόπος

Πρέπει λοιπόν ο αριθμός 2n 3 να είναι τέλειο τετράγωνο και επειδή είναι περιττός

(ως άθροισμα άρτιου και περιττού), θα πρέπει να είναι τέλειο τετράγωνο περιττού.

Άρα υπάρχει k φυσικός αριθμός ώστε να είναι

2 22n 3 (2k 1) n 2k 2k 1 . Άρα

2 2 225 25A [2k 2k 1 2 (2k 1) ] A (2k 2k 1 2 2k 1)

2 2

2 2A 25k (5k) , και η απόδειξη ολοκληρώθηκε.

Γ τρόπος

Προφανώς το 2n 3 είναι περιττός φυσικός, οπότε

2

2n 3 1A 5

2

με

2n 3 1

2

φυσικό.

ΘΕΜΑ 250 (ΔΗΜΗΤΡΗΣ ΙΩΑΝΝΟΥ )

Αν x,y και αν ισχύει ότι: 2 2 2 2x y 3x y 8(x y 2xy) 43 , να βρείτε την

αριθμητική τιμή της παράστασης: 4 4A x y .

Λύση:

Σε τέτοιες ασκήσεις που δίνεται εξίσωση στους πραγματικούς και ζητάει να

υπολογιστεί μια παράσταση όπως στην περίπτωση μας με ακριβές νούμερο τότε

προσπαθούμε να την φέρουμε σε μορφή 2 2 2

1 1 2 2 k ka x a x ....a x 0 .

Page 50: ΠΡΟΤΕΙΝΟΜΕΝΕΣ ΑΣΚΗΣΕΙΣ ΓΙΑ ΜΑΘΗΤΙΚΟΥΣ ΔΙΑΓΩΝΙΣΜΟΥΣ 201 - 300

http://www.mathematica.gr/forum/viewtopic.php?f=109&t=15584

Επιμέλεια: xr.tsif Σελίδα 50

και i

a 0 οπότε θα έχει λύση μόνο για i

x 0 με i 1,2,...k .

Στην περίπτωση μας τώρα μετά από πράξεις η εξίσωση γίνεται:

2 2(x y 4) 3(xy 3) 0 άρα όπως είπαμε παραπάνω θα έχει λύση μόνο για

x y 4 , xy 3 .

Οπότε 2 2x y 10 και 4 4

x y 82 .

ΘΕΜΑ 251 (Socrates)

Σε ένα ενυδρείο υπάρχουν ψάρια και των δύο φύλων. Επιλέγουμε στη τύχη δύο

ψάρια. Αν η πιθανότητα να είναι και τα δύο του ιδίου φύλου είναι 1

2 , να δείξετε ότι

ο αριθμός των ψαριών στο ενυδρείο είναι τέλειο τετράγωνο ακεραίου.

Λύση:

Έστω t, τα αρσενικά και τα θηλυκά αντίστοιχα τότε C(t,2) C( ,2) 1

C(t,2) C( ,2) t 2

.

Μετά από πράξεις καταλήγουμε ότι 2(t ) t άρα αποδείχθηκε.

Αφού t, ακέραιοι τότε οι τιμές των t, ανήκουν στο σύνολο

k(k 1) k(k 1) (k 2)(k 1)A , ,

2 2 2

.

ΘΕΜΑ 252 (Socrates)

Αν x και y είναι μη αρνητικοί ακέραιοι, τότε ο αριθμός x y είναι επίσης μη

αρνητικός ακέραιος. Η πράξη ικανοποιεί τη συνθήκη: (x y)(y z) x z (1).

Αν 23 47 0 να υπολογίσετε τον αριθμό 61 89 .

Page 51: ΠΡΟΤΕΙΝΟΜΕΝΕΣ ΑΣΚΗΣΕΙΣ ΓΙΑ ΜΑΘΗΤΙΚΟΥΣ ΔΙΑΓΩΝΙΣΜΟΥΣ 201 - 300

http://www.mathematica.gr/forum/viewtopic.php?f=109&t=15584

Επιμέλεια: xr.tsif Σελίδα 51

Λύση:

Από υπόθεση έχουμε: 61 89 μη αρνητικός ακέραιος.

Αν y z , τότε

(1) (x z)(z z) (x z) x z 0   ή z z 1

το πρώτο απορρίπτεται αφού 23 47 0 ,άρα z z 1 .

Αν z x , τότε

 (1) (x y)(y x) 1 (61 89)(89 61) 1 με 61 89 0 , 89 61 0 μη

αρνητικοί ακέραιοι με γινόμενο ίσο 1 ,άρα 61 89 1 .

ΘΕΜΑ 253 (Socrates)

Ένας αριθμός ατόμων παίζουν το παρακάτω παιχνίδι: Κάθε παίκτης έχει αρχικά

300 €. Στην αρχή κάθε γύρου, ο κάθε παίκτης δίνει 10 € στη "μάνα". Τα χρήματα

αυτά δεν επιστρέφουν ξανά στο παιχνίδι. Στο τέλος κάθε γύρου, κάποιος παίκτης

(π.χ. ο χαμένος ) μοιράζει τα χρήματά του στους υπόλοιπους παίκτες σε ίσα ποσά και

αποχωρεί από το παιχνίδι. Το παιχνίδι τελειώνει όταν μείνει μόνο ένας παίκτης, ο

οποίος είναι και ο νικητής. Αν ο νικητής έχει, στο τέλος του παιχνιδιού, τόσα

χρήματα όσα ξεκίνησε, δηλαδή 300 €, να βρείτε τον αρχικό αριθμό των παικτών.

Λύση:

Έστω n οι παίκτες, οπότε παίχτηκαν n 1 γύροι. Όλα τα λεφτά ήταν (υπήρχαν ...)

300 n τα οποία είναι ίσα με αυτά που κράτησε η μάνα, και έφυγαν από το παιχνίδι,

συν τα 300 του νικητή:

n·10 (n 1)10 (n 2)10 ... 2·10 300 n·300

2n(n 1)10 290 300n n 59n 58 0 n 1,n 58

2

, κ.λπ.

Page 52: ΠΡΟΤΕΙΝΟΜΕΝΕΣ ΑΣΚΗΣΕΙΣ ΓΙΑ ΜΑΘΗΤΙΚΟΥΣ ΔΙΑΓΩΝΙΣΜΟΥΣ 201 - 300

http://www.mathematica.gr/forum/viewtopic.php?f=109&t=15584

Επιμέλεια: xr.tsif Σελίδα 52

ΘΕΜΑ 254 (ΔΗΜΗΤΡΗΣ ΙΩΑΝΝΟΥ )

Να αποδείξετε ότι αν pπρώτος, με p 3 , τότε ο αριθμός 2p 1 διαιρείται με το 24 .

Λύση:

Έχουμε ότι 2p 1 (p 1)(p 1) . Επειδή οι αριθμοί (p 1),(p 1) είναι δύο

διαδοχικοί άρτιοι (αφού p 2 ), έπεται εύκολα ότι 8 / (p 1)(p 1) .

Επίσης, οι (p 1),p,(p 1) είναι 3 διαδοχικοί φυσικοί αριθμοί. Κάποιος λοιπόν από

αυτούς θα είναι πολλαπλάσιο του 3 . Επειδή p πρώτος με p 3 , κάποιος από τους

(p 1),(p 1) θα είναι πολλαπλάσιο του 3 .

Άρα, 3 / (p 1)(p 1) .

Οπότε, αφού (3,8) 1 , έχουμε ότι ο αριθμός 3 8 / (p 1)(p 1) .

Β τρόπος

μπορούμε να δείξουμε το ζητούμενο για κάθε p περιττό και μη διαιρετό από το 3

(και όχι απαραίτητα πρώτο):

Είναι 2 2p 1(mod8) p 1 0(mod8)   (1) , αφού p περιττός, και

2 2p 1(mod3) p 1 0(mod3) (2) ,αφού p 0(mod3)

Το ζητούμενο έπεται απο τις (1) και (2).

Γ τρόπος

Επειδή ο pείναι πρώτος, θα έχει την μορφή p 6k 1 ή p 6k 5

1η ΠΕΡΙΠΤΩΣΗ: 2p 6k 1 p 1 12k(3k 1)

* Αν k άρτιος, δηλ. k 2n τότε 2p 1 24n(6n 1) και άρα έχουμε το ζητούμενο.

Page 53: ΠΡΟΤΕΙΝΟΜΕΝΕΣ ΑΣΚΗΣΕΙΣ ΓΙΑ ΜΑΘΗΤΙΚΟΥΣ ΔΙΑΓΩΝΙΣΜΟΥΣ 201 - 300

http://www.mathematica.gr/forum/viewtopic.php?f=109&t=15584

Επιμέλεια: xr.tsif Σελίδα 53

* Αν k περιττός, δηλ. k 2n 1 τότε 2

p 1 12(2n 1)(6n 4) 24(2n 1)(3n 2) οπότε και πάλι έχουμε το ζητούμενο.

2η ΠΕΡΙΠΤΩΣΗ: p 6k 5

Εργαζόμαστε ομοίως.

ΘΕΜΑ 255 (ΔΗΜΗΤΡΗΣ ΙΩΑΝΝΟΥ )

Στο επίπεδο θεωρούμε ένα 2011– γωνο. Να εξετάσετε αν είναι δυνατόν να φέρουμε

μια ευθεία πάνω σε αυτό το επίπεδο που να τέμνει όλες τις πλευρές του.

Λύση:

Ας υποθέσουμε ότι υπάρχει τέτοια ευθεία.

Ονομάζουμε το πολύγωνο 1 2 2011

A A ...A . Τα σημεία 1

A και 2

A θα ανήκουν σε

διαφορετικά ημιεπίπεδα ως προς την ευθεία. Το 3

A όπως επίσης και όλα τα σημεία

με περιττό δείκτη θα ανήκουν στο ίδιο ημιεπίπεδο. Όμως το τμήμα 1 2011

A A είναι

πλευρά του πολυγώνου, άρα τα 1 2011

A ,A ανήκουν σε διαφορετικά ημιεπίπεδα ως

προς την ευθεία, άτοπο.

ΘΕΜΑ 256 (ΔΗΜΗΤΡΗΣ ΙΩΑΝΝΟΥ )

Αν για τους πραγματικούς αριθμούς a,b,c ισχύει:

2 2ab ac b ac c ab και οι ρίζες της εξίσωσης

2ax bx c 0 είναι

πραγματικές, τότε να αποδείξετε μόνο η μία ρίζα της εξίσωσης θα περιέχεται μεταξύ

0 και 2 (δηλαδή μόνο μια ρίζα της θα ανήκει στο διάστημα (0,2) .

Λύση:

Λόγω της ιδιότητας | x | | y | | x y |, ισχύει

Page 54: ΠΡΟΤΕΙΝΟΜΕΝΕΣ ΑΣΚΗΣΕΙΣ ΓΙΑ ΜΑΘΗΤΙΚΟΥΣ ΔΙΑΓΩΝΙΣΜΟΥΣ 201 - 300

http://www.mathematica.gr/forum/viewtopic.php?f=109&t=15584

Επιμέλεια: xr.tsif Σελίδα 54

2 2 2 2| a || b c | | b ac | | c ab | | b c ab ac | | b c || a b c |, οπότε είναι

| a | | a b c | , άρα b c

1 1a a

| | .

Από τους τύπους Vieta, αυτή γράφεται ως 1 2 1 2

|1 (x x ) x x | 1 , δηλαδή

1 2| x 1 || x 1 | 1 . Τότε, μία τουλάχιστον από τις παρενθέσεις είναι 1

Ας είναι π.χ. 1

| x 1 | 1 , από εδώ βρίσκουμε 1

x (0,2) .

Θα αποδείξουμε τώρα, ότι η άλλη ρίζα πρέπει να είναι μη θετική.

Ας είναι 1 2

as x x

b και

1 2

cp x x

a . Αν ήταν

2x 0 , θα είχαμε s,p 0 . Με

χρήση των s,p η αρχική συνθήκη γράφεται ως

2 2 2 2 2 2 2 2| a s a p | | a s a p | | a p a s | , δηλαδή 2 2

| s p | | s p | | p s | . Λόγω της

υπόθεσης και της ανισότητας 2s 4p p , όλα μέσα στις απόλυτες τιμές είναι θετικά,

οπότε λαμβάνουμε

2 2 2 2 2 2s p s p p s 2p s p 2p 4p p p 2p 0 , άτοπο.

Β τρόπος

Αρκεί να δειχτεί ότι: f (0)f (2) 0 c(4a 2b c) 0 (1).

Μπορούμε να υποθέσουμε a 0 , διαφορετικά αλλάζουμε τα πρόσημα κ.λπ. Είναι 2

b 4ac , οπότε και 2

b ac 0 , αφού 2 2

b ac 0 b ac 4ac , άτοπο.

Είναι 2 2 2 2

ab ac b ac c ab a b c b ac c ab

a b c (b c)(a b c) a a b c a

2a b c 0, (2) ( 2a b c 0)

και 2 2 2 2

ab ac b ac ab ac b ac ab ac b ac ab a c b ac

Page 55: ΠΡΟΤΕΙΝΟΜΕΝΕΣ ΑΣΚΗΣΕΙΣ ΓΙΑ ΜΑΘΗΤΙΚΟΥΣ ΔΙΑΓΩΝΙΣΜΟΥΣ 201 - 300

http://www.mathematica.gr/forum/viewtopic.php?f=109&t=15584

Επιμέλεια: xr.tsif Σελίδα 55

2 2ab b , (3) b ab 2ac, (4) .

Αν αληθεύει η (3) , τότε b 0 a b (2) c 0 και η (1) αληθεύει, αφού

c(2a b (2a b c)) 0 .

Αν αληθεύει η (4) , τότε ab ac 0 b c , και

2b ab 2ac 4ac ab 2ac 2c b 0 c 0 ,

και η (1) αληθεύει, αφού c(2(2a b c) c) 0 .

ΘΕΜΑ 257 (Cretanman )

Να αποδείξετε ότι ο αριθμός 2n n 1 δεν διαιρείται από το 25 .

Λύση:

Αν συνέβαινε αυτό τότε το 100 θα έπρεπε να διαιρεί τον 2 24n 4n 4 (2n 1) 3 ,

που σημαίνει ότι ο αριθμός 2 24n 4n 4 (2n 1) 3 λήγει σε δύο μηδενικά.

Εξετάζουμε λοιπόν το τελευταίο ψηφίο του αριθμού 2 24n 4n 4 (2n 1) 3 .

Κάθε περιττός στο τετράγωνο έχει τελευταίο ψηφίο 1,5,9 αν τώρα προσθέσουμε 3

προκύπτει ότι το τελευταίο ψηφίο του 2 24n 4n 4 (2n 1) 3 θα είναι 4 ή 8 ή

2 άρα όχι 0 .

Οπότε το 25 δεν διαιρεί τον αριθμό .

Β τρόπος

Θα αποδείξουμε, ότι ο αριθμός 2

n n 1 δεν διαιρείται με το 5 .

Ας είναι n 5k u , με u 0,1,2,3,4 . Τότε, 2 2 2

n n 1 25k 10ku 5k u u 1 .

Απομένει να διαπιστώσουμε ότι το 2

u u 1 δεν διαιρείται με το 5 .

Page 56: ΠΡΟΤΕΙΝΟΜΕΝΕΣ ΑΣΚΗΣΕΙΣ ΓΙΑ ΜΑΘΗΤΙΚΟΥΣ ΔΙΑΓΩΝΙΣΜΟΥΣ 201 - 300

http://www.mathematica.gr/forum/viewtopic.php?f=109&t=15584

Επιμέλεια: xr.tsif Σελίδα 56

Πράγματι, έχουμε 2

1,u 0

3,u 1

u u 1 7,u 2

13,u 3

21,u 4.

.

ΘΕΜΑ 258 (ΔΗΜΗΤΡΗΣ ΙΩΑΝΝΟΥ )

Θεωρούμε ένα κυρτό 100 – γωνο 1 2 100

A A ...A . Φέρνουμε την διαγώνιο 42 81

A A που το

χωρίζει σε δύο κυρτά πολύγωνα A,B . Πόσες κορυφές και πόσες διαγώνιες έχει

καθένα από τα δύο αυτά πολύγωνα;

Λύση:

Το πολύγωνο A θα έχει 42 20 62 κορυφές και το πολύγωνο B θα έχει

81 42 1 40 κορυφές.

Οι διαγώνιοι σε ένα πολύγωνο με n κορυφές ενώνουν κάθε κορυφή με όλες τις

υπόλοιπες κορυφές εκτός αυτές που βρίσκονται εκατέρωθεν της κορυφής. Από κάθε

κορυφή ενός πολυγώνου φέρνουμε n 3 ευθύγραμμα τμήματα (διαγώνιες).

Άρα το συνολικό πλήθος των διαγωνίων είναι n(n 3)

2

, γιατί έχουμε μετρήσει δύο

φορές την καθεμία (την ΑΓ και την ΓΑ ).

Έτσι το A έχει 62(62 3) 62 59

31 59 18292 2

διαγώνιες και το B

40(40 3) 40 3720 37 740

2 2

διαγώνιες.

Page 57: ΠΡΟΤΕΙΝΟΜΕΝΕΣ ΑΣΚΗΣΕΙΣ ΓΙΑ ΜΑΘΗΤΙΚΟΥΣ ΔΙΑΓΩΝΙΣΜΟΥΣ 201 - 300

http://www.mathematica.gr/forum/viewtopic.php?f=109&t=15584

Επιμέλεια: xr.tsif Σελίδα 57

ΘΕΜΑ 259 (ΔΗΜΗΤΡΗΣ ΙΩΑΝΝΟΥ )

Να λυθεί η εξίσωση: x x 2 327 27 1 3cos x

.

(ΣΗΜΕΙΩΣΗ: Με το σύμβολοcosx συμβολίζουμε το συνx ).

Λύση:

Πράγματι έχουμε:

Από την εξίσωση και για xt 27 ισοδύναμα παίρνουμε:

2 2 3 2 3t 1 3cos x t 1 0, Δ 0 ... cos x 0 x 0...

Β τρόπος

Το α' μέλος είναι 3 , ενώ το β' 3 .....

x x27 27 1 3

2 33 3cos x

Άρα : x x27 27 1 3

και παίρνω x 0 .

ΑΝΑΛΥΤΙΚΑ

Έχουμε ότι: 2

2 3 2 3 33cos x 3 cos x 3 cosx 3 1 3 (1).

Επίσης από την ταυτότητα του Euler έχουμε:

x 3 x 3 3 x x(3 ) (3 ) 1 3 3 3 1

x x x x 2 x 2 x 21(3 3 1)[(3 3 ) (3 1) (3 1) ] 0

2

x x x x27 27 1 3 0 27 27 1 3

(2).

Από τις σχέσεις (1) και (2) έχουμε ότι:

x x27 27 1 3

(*) και 2 3

3cos x 3 (**)

Page 58: ΠΡΟΤΕΙΝΟΜΕΝΕΣ ΑΣΚΗΣΕΙΣ ΓΙΑ ΜΑΘΗΤΙΚΟΥΣ ΔΙΑΓΩΝΙΣΜΟΥΣ 201 - 300

http://www.mathematica.gr/forum/viewtopic.php?f=109&t=15584

Επιμέλεια: xr.tsif Σελίδα 58

Από την (*) έχουμε ότι:

x x 2 x x 2 x

x

127 1 3 (27 ) 1 2 27 0 (27 1) 0 27 1 x 0

27 .

Εύκολα τώρα διαπιστώνουμε ότι η x 0 επαληθεύει τόσο την (**) όσο και την

δοσμένη εξίσωση και άρα είναι η λύση της.

ΘΕΜΑ 260 (ΔΗΜΗΤΡΗΣ ΙΩΑΝΝΟΥ )

Αν πολλαπλασιάσουμε δύο οποιουσδήποτε αριθμούς του συνόλου S {2,5,13}

και ύστερα αφαιρέσουμε τον αριθμό 1 , τότε ο νέος αριθμός που θα προκύψει θα

είναι τέλειο τετράγωνο. Να αποδείξετε ότι το σύνολο S δεν μπορεί να επεκταθεί με

την προσθήκη ενός άλλου ακεραίου χωρίς να παραβιασθεί η παραπάνω συνθήκη.

Λύση:

Έστω ότι υπάρχει τέτοιο q , τότε υπάρχουν ακέραιοι x,y,z ώστε

2 2 22q 1 x ,5q 1 y ,13q 1 z .

Από την πρώτη, φαίνεται ότι x περιττός. Είναι ακόμα 2 2 2x z 3y 1 (I).

Από εδώ βλέπουμε, ότι οι y,z είναι και οι δύο περιττοί ή και οι δύο άρτιοι.

Στην πρώτη περίπτωση, θέτοντας στην (Ι) x 2k 1,y 2l 1,z 2m 1 ,

βρίσκουμε 2 2 24k 4k 4m 4m 12 12 2, δηλαδή 4 / 2 , άτοπο.

Ας είναι, λοιπόν, y 2 ,z 2m .

Επειδή είναι 2 25z 13y 8 , βρίσκουμε

2 25 13m 2 . Από εδώ είναι ,m άρτιοι ή

,m περιττοί.

Αν 2p,m 2r , βρίσκουμε 2 24(5p 13r ) 2 , άτοπο.

Page 59: ΠΡΟΤΕΙΝΟΜΕΝΕΣ ΑΣΚΗΣΕΙΣ ΓΙΑ ΜΑΘΗΤΙΚΟΥΣ ΔΙΑΓΩΝΙΣΜΟΥΣ 201 - 300

http://www.mathematica.gr/forum/viewtopic.php?f=109&t=15584

Επιμέλεια: xr.tsif Σελίδα 59

Αν 2p 1,m 2r 1 , βρίσκουμε 2 220p 20p 52r 52r 10 , δηλαδή 4 / 10 ,

πάλι άτοπο.

Για την ιστορία, είναι το πρόβλημα 1 της διεθνούς ολυμπιάδας το 1986!

ΘΕΜΑ 261 (Socrates)

Σε ένα μαγικό τετράγωνο 4x4 το άθροισμα των αριθμών κάθε στήλης, γραμμής και

διαγωνίου είναι το ίδιο και έστω ίσο με s . Να δείξετε ότι το άθροισμα των αριθμών

στα τετράγωνα των τεσσάρων κορυφών του τετραγώνου είναι επίσης ίσο με s .

Λύση:

2(A D E F) (A B C D) (E F G H) (A a d H)

(E c b D) (B a c F) (C b d G) s s s s s s 2s .

Με παρόμοιο τρόπο δείχνουμε π.χ.

B C F G a b c d s και, εννοείται,

ισχύει το ίδιο για τα "συμμετρικά" τους

τετραγωνάκια.

Β τρόπος

Ονομάζουμε τα τετραγωνάκια ως εξής:

1 2 3 4a   a   a   a

1 2 3 4b   b b b

1 2 3 4c   c c   c

1 2 3 4d   d d d

Θέλουμε να δείξουμε ότι: 1 4 1 4

a a d d s  . Το άθροισμα όλων των αριθμών

είναι ίσο με 4s . Επίσης είναι 2 2 2 2 3 3

a b c d a ... d 2s , άρα αρκεί να

δείξουμε ότι: 1 1 4 4

b c b c s (1).

Page 60: ΠΡΟΤΕΙΝΟΜΕΝΕΣ ΑΣΚΗΣΕΙΣ ΓΙΑ ΜΑΘΗΤΙΚΟΥΣ ΔΙΑΓΩΝΙΣΜΟΥΣ 201 - 300

http://www.mathematica.gr/forum/viewtopic.php?f=109&t=15584

Επιμέλεια: xr.tsif Σελίδα 60

Αρκεί τώρα ,να παρατηρήσουμε ότι αφού οι 2 διαγώνιοι έχουν άθροισμα 2s θα είναι

2 3 3 4 1 1 4 4a a d d b c b c 2s , άρα αντί της (1) αρκεί να δείξουμε ότι

2 3 3 4 1 1 4 4a a d d b c b c , το οποίο ισχύει αφού είναι:

2 3 3 4 2 3 2 3a a d d 2s (b b c c ) και

1 1 4 4 2 3 2 3b c b c 2s (b b c c ) .

ΘΕΜΑ 262 (Socrates)

Να βρείτε τις ακέραιες λύσεις της εξίσωσης 11

x 2x 1 3y 4y 1 25

.

Λύση:

Επειδή ο 4y 1 είναι ισότιμος με 3mod4 , δεν είναι ποτέ τέλειο τετράγωνο. Άρα ο

4y 1 είναι άρρητος. Συνεπώς η ισότητα 11

x 2x 1 3y 4y 1 25

δείχνει

ότι και ο 2x 1 είναι άρρητος. Από αυτό και την δοθείσα εύκολα βλέπουμε ότι

ισχύει 11x 3y 2 2x 1 4y 1

5

(*). Λύνοντας το σύστημα των

εξισώσεων αυτών, ισοδύναμα 11

x 3y 2 2x 1 4y 15

, θα βρούμε

x 5,y 3 .

(*) Αυτό βασίζεται στο εξής:

Έστω p,q ρητοί και a, b άρρητες ρίζες ακεραίων με p a q b .

Υψώνοντας στο τετράγωνο την (p q) a b , εύκολα διαπιστώνουμε ότι ισχύει

(p q) ( a b) .

ΘΕΜΑ 263 (Socrates)

Αν m,n N {0,1} τέτοιοι ώστε m

6 0n

, να δείξετε ότι m 1

6n 2mn

.

Page 61: ΠΡΟΤΕΙΝΟΜΕΝΕΣ ΑΣΚΗΣΕΙΣ ΓΙΑ ΜΑΘΗΤΙΚΟΥΣ ΔΙΑΓΩΝΙΣΜΟΥΣ 201 - 300

http://www.mathematica.gr/forum/viewtopic.php?f=109&t=15584

Επιμέλεια: xr.tsif Σελίδα 61

Λύση:

Η ζητούμενη είναι πιο ισχυρή από τη δοσμένη ανισότητα (δηλαδή η δοσμένη

προκύπτει αμέσως από την ζητούμενη) αλλά εδώ έχουμε να κάνουμε με φυσικούς

αριθμούς και αυτό ίσως μας βοηθήσει. Για φυσικούς ισχύει το εξής ωραίο.

Αν x y τότε x y 1 που είναι πιο ισχυρό. Θα προσπαθήσουμε να το

χρησιμοποιήσουμε. Κοιτάμε την δοσμένη. Την γράφουμε m

6n

. Υψώνουμε στο

τετράγωνο (για να έχουμε μόνο φυσικούς αριθμούς) και παίρνουμε 2 26n m .

Άρα 2 26n m 1 (*). Τώρα κοιτάμε την ζητούμενη.

Την πάμε στη ισοδύναμη μορφή 2

2 2 2 2

m 1 16

n 4m n n .

Άρα βλέπουμε ότι είναι πάλι πιο ισχυρή από την (*) .

Μήπως μπορεί η (*) να βελτιωθεί κι άλλο? Αν ήταν λίγο πιο ισχυρή θα λύναμε την

άσκηση αφού ,αν προς στιγμήν ξέραμε ότι 2 26n m 2 (**) , τότε

2 2

2 2 2 2 2

m 2 m 1 16

n n 4m n n

(αυτή η τελευταία βγαίνει εύκολα με πράξεις).

Μένει λοιπόν να δείξουμε την (**). Αν κοιτάξουμε την (*) βλέπουμε ότι πρέπει να

δείξουμε ότι 2 26x y 1 για όλους τους φυσικούς x,y .

Απόδειξη του τελευταίου:

Έστω ότι 2 26x y 1 . Το πρώτο μέλος είναι πολλαπλάσιο του 6 . Θέτουμε

y 6a b με b 0,1,2,3,4,5 .

Τότε το δεύτερο μέλος γίνεται πολλαπλάσιο του 6 συν 1,2,5,4,5,2 δηλαδή όχι

πολλαπλάσιο του 6 , άτοπο.

Άρα δεν υπάρχουν τέτοιοι φυσικοί. Άρα ισχύει η (**) που λύνει την άσκηση όπως

είδαμε.

Page 62: ΠΡΟΤΕΙΝΟΜΕΝΕΣ ΑΣΚΗΣΕΙΣ ΓΙΑ ΜΑΘΗΤΙΚΟΥΣ ΔΙΑΓΩΝΙΣΜΟΥΣ 201 - 300

http://www.mathematica.gr/forum/viewtopic.php?f=109&t=15584

Επιμέλεια: xr.tsif Σελίδα 62

Β τρόπος

Νομίζω βρήκα μία λύση με CS.

Αρχικά έχουμε ότι n m 1 και m 6n .

Θέλουμε να δείξουμε ότι

2

m 16

n 2mn

ή 2 2 2 224m n (2m 1) .

Από CS έχουμε ότι 2 2 3(2m 1) (m 1)(4m 1) . Ισχύει

4m 4 6n 4m 1 4 6n και 3 2 2m 1 (m 1)(m m 1) 6n(m m 1) .

Άρα, έχουμε δείξει ότι 2 2 2 2 2(2m 1) 4 6n· 6n·(m m 1) 24n (m m 1)

Οπότε, αρκεί να δείξουμε ότι 2 2 2 224n (m m 1) 24n m ή

2 2m m 1 m m 1 , που ισχύει από την υπόθεση.

ΘΕΜΑ 264 (ΔΗΜΗΤΡΗΣ ΙΩΑΝΝΟΥ )

Το υποσύνολο A των πραγματικών αριθμών, έχει τις ιδιότητες:

Z A

2 3  A  

Αν a,b  A a b   A , ab  A . Να αποδείξετε ότι 1

  A2 3

.

(όπου Z είναι το σύνολο των ακεραίων)

Λύση:

Παρατηρούμε πρώτα ότι 1

3 22 3

. Άρα αρκεί να δείξουμε ότι ο

τελευταίος αριθμός είναι στοιχείο του συνόλου A .

Τώρα 2 3  A   οπότε ( 2 3)( 2 3) A , δηλαδή 5 2 6 A .

Page 63: ΠΡΟΤΕΙΝΟΜΕΝΕΣ ΑΣΚΗΣΕΙΣ ΓΙΑ ΜΑΘΗΤΙΚΟΥΣ ΔΙΑΓΩΝΙΣΜΟΥΣ 201 - 300

http://www.mathematica.gr/forum/viewtopic.php?f=109&t=15584

Επιμέλεια: xr.tsif Σελίδα 63

Επίσης 5 A , αφού είναι ακέραιος. Άρα (5 2 6) ( 5) 2 6 A .

Τότε (2 6)( 2 3) 6 2 4 3 A .

Τώρα ( 5)( 2 3) 5 2 5 3 A .

Άρα (6 2 4 3) ( 5 2 5 3) 2 3 A .

Τέλος αφού 1 A έχουμε ( 1)( 2 3) 3 2 A , όπως θέλαμε.

ΘΕΜΑ 265 (ΔΗΜΗΤΡΗΣ ΙΩΑΝΝΟΥ )

Σε ένα ισοσκελές τρίγωνο ABC θεωρούμε ένα σημείο D στην βάση BC και ένα

σημείο E στην πλευρά ACέτσι ώστε να είναι γωνία B A D 2 CDE

. Να

αποδείξετε ότι το τρίγωνο ADE είναι ισοσκελές. (Για την Γ Γυμνασίου)

Λύση:

Αφού το τρίγωνο ABC είναι ισοσκελές Β C φ

.Στο τρίγωνο ADC η ω

είναι

εξωτερική , άρα ο ο

ω Α 2x φ 180 2φ 2x φ 180 φ 2x

(1).

Αλλά (1)

ο ο oβ 180 ω x β 180 180 φ 2x x φ x

και η α

είναι εξωτερική στο EDC , άρα α φ x

, δηλαδή β α

.

Page 64: ΠΡΟΤΕΙΝΟΜΕΝΕΣ ΑΣΚΗΣΕΙΣ ΓΙΑ ΜΑΘΗΤΙΚΟΥΣ ΔΙΑΓΩΝΙΣΜΟΥΣ 201 - 300

http://www.mathematica.gr/forum/viewtopic.php?f=109&t=15584

Επιμέλεια: xr.tsif Σελίδα 64

ΘΕΜΑ 266 (ΔΗΜΗΤΡΗΣ ΙΩΑΝΝΟΥ )

Θεωρούμε τα πολυώνυμα 4 3 2P(x) x 3x x 3,Q(x) x 2x 3 και

2R(x) x 5x a .

α) Να ορίσετε το a έτσι ώστε το πολυώνυμο R(x) να διαιρείται από το x 2 .

β) Να αναλύσετε σε γινόμενο παραγόντων τα πολυώνυμα R(x),P(x),Q(x) .

γ) Να αποδείξετε ότι η παράσταση 2 P(x)x x 15

Q(x) είναι τέλειο τετράγωνο.

(Για την Γ Γυμνασίου)

(ΣΗΜΕΙΩΣΗ: Η διαίρεση πολυωνύμου διά πολυώνυμο δεν είναι μέσα στην

διδακτέα ύλη, νομίζω όμως ότι όποιος λαβαίνει μέρος σε τέτοιου είδους

διαγωνισμούς, πρέπει να την μελετήσει.)

Λύση:

α) Πρέπει το x 2 να είναι ρίζα του R(x) . Άρα έχουμε 2

2 5 2 a 0 a 14 .

β) 4 3 3 3P(x) x 3x x 3 x (x 3) (x 3) (x 3)(x 1)

2(x 1)(x 3)(x x 1) .

2 2Q(x) x 2x 3 x 2x 2 1 (x 1)(x 1) 2(x 1) (x 1)(x 3) .

2 2R(x) x 5x 14 (x 5x 14) (x 2)(x 7) .

γ) 2

2 2P(x) (x 1)(x 3)(x x 1)x x 15 x x 15

Q(x) (x 1)(x 3)

2 2 2x x x x 1 15 16 4 .

Page 65: ΠΡΟΤΕΙΝΟΜΕΝΕΣ ΑΣΚΗΣΕΙΣ ΓΙΑ ΜΑΘΗΤΙΚΟΥΣ ΔΙΑΓΩΝΙΣΜΟΥΣ 201 - 300

http://www.mathematica.gr/forum/viewtopic.php?f=109&t=15584

Επιμέλεια: xr.tsif Σελίδα 65

ΘΕΜΑ 267 (ΔΗΜΗΤΡΗΣ ΙΩΑΝΝΟΥ )

α) Αν 2 2 2b c a ,b c να υπολογίσετε την παράσταση:

3 3 3 3b c b c

b c b c

.

β) Αν 1

a k,a 0,a

να βρεθεί η παράσταση: 4

4

1a

a σαν έκφραση του k .

(Για την Γ Γυμνασίου)

Λύση:

α) 3 3 3 3 3 3 3 3

2 2

b c b c (b c )(b c) (b c )(b c)

b c b c b c

2 2 2 2

2 2

[(b c)(b bc c )(b c)] [(b c)(b bc c )(b c)]

b c

2 2 2 2 2 2 2 2

2 2

(b c )(b bc c ) (b c )(b bc c )

(b c )

2 2 2 2 2 2

2 2 2 2 2

2 2

(b c )[(b bc c ) (b bc c )](b bc c b bc c ) 2a

(b c )

.

Σημείωση: χρησιμοποίησα τις ταυτότητες:

3 3 2 2a b (a b)(a ab b ) και 3 3 2 2

a b (a b)(a ab b ) .

β) 2 2 2 2 2 2

2 2

1 1 1 1a k (a ) k a 2 k a k 2

a a a a .

2 2 2 2 2 2 4 4 2

2 2 4

1 1 1a k 2 (a ) (k 2) a 2 k 4k 4

a a a

4 4 2

4

1a k 4k 2

a .

Page 66: ΠΡΟΤΕΙΝΟΜΕΝΕΣ ΑΣΚΗΣΕΙΣ ΓΙΑ ΜΑΘΗΤΙΚΟΥΣ ΔΙΑΓΩΝΙΣΜΟΥΣ 201 - 300

http://www.mathematica.gr/forum/viewtopic.php?f=109&t=15584

Επιμέλεια: xr.tsif Σελίδα 66

ΘΕΜΑ 268 (ΔΗΜΗΤΡΗΣ ΙΩΑΝΝΟΥ )

Σε τρίγωνο ABC τα μήκη των πλευρών του, είναι διαδοχικοί ακέραιοι και ισχύει

AB BC CA . Αν η διχοτόμος AD είναι κάθετη στην διάμεσο BE , να βρεθούν τα

μήκη των πλευρών του τριγώνου. (Για την Β Γυμνασίου)

Λύση:

Αφού τα μήκη των πλευρών του, είναι διαδοχικοί ακέραιοι και ισχύει

AB BC CA , έστω AB x , BC x 1 , AC x 2 . Στο τρίγωνο ABE η

διχοτόμος AZ είναι και ύψος , άρα το τρίγωνο είναι ισοσκελές. Άρα

AB AE EC δηλαδή x 2 2x x 2 . Άρα AB 2 , BC 3 , AC 4 .

ΘΕΜΑ 269 (ΔΗΜΗΤΡΗΣ ΙΩΑΝΝΟΥ )

Να βρείτε τους πραγματικούς αριθμούς 1 2 3 4

x ,x ,x ,x για τους οποίους ισχύουν

ταυτόχρονα οι σχέσεις 2 2 2 2

1 2 3 4x x x x 1 και 4 4 4 4

1 2 3 4x x x x 1 .

(Για την Γ Γυμνασίου)

Page 67: ΠΡΟΤΕΙΝΟΜΕΝΕΣ ΑΣΚΗΣΕΙΣ ΓΙΑ ΜΑΘΗΤΙΚΟΥΣ ΔΙΑΓΩΝΙΣΜΟΥΣ 201 - 300

http://www.mathematica.gr/forum/viewtopic.php?f=109&t=15584

Επιμέλεια: xr.tsif Σελίδα 67

Λύση:

2 2 2 2 2 2 2 2

1 2 3 4 1 2 3

2

4) ( )x x x x 1 [ x ]( x x 1x

2 2 2 2 2 2 2 2

1 2 1

2 2

2

4 4

3 4 4 23

2 2

1 1 2) 2 )( ) ( )(x x (x x x x x x 1 x x 2x x

4 4 2 2 2 2 2 2

3 4 3 4

2 2 2 2 2 2 2 2

1 2 3 4 1 2 3 41 2 3 42 )( ) x x 2x x 1 2x x 2 )((x x x x (x x x x ) 2x x 0 .

Πρέπει 2 2

1 2x x 0 και 2 2 2 2

1 2 3 4(x x x( 0x) ) και 2 2

3 4x x 0 και άρα

1 2 3 4x x x x 0 , Άτοπο.

Άρα δεν υπάρχουν 1 2 3 4

x ,x ,x ,x που ικανοποιούν τις σχέσεις.

ΘΕΜΑ 270 (ΔΗΜΗΤΡΗΣ ΙΩΑΝΝΟΥ )

Αν x x

x x

a ay ,a 0,a 1,x 0

a a

και αν

4x 4x

4x 4x

a az

a a

, να εκφράσετε το z σαν

συνάρτηση του y . (Για την Γ Γυμνασίου)

Λύση:

x x 2x

2y 1

x 2x 2x 2x

x x 2x

a a a 1y ya a a a

a a a 1

y 1y 1 (y 1) y 1

y 1

4

4x 4x 8x 4 44

44x 4x 8x 4 4

4

(y 1)1

a a a 1 (y 1) (y 1)(y 1)z

(y 1)a a a 1 (y 1) (y 1)1

(y 1)

.

Page 68: ΠΡΟΤΕΙΝΟΜΕΝΕΣ ΑΣΚΗΣΕΙΣ ΓΙΑ ΜΑΘΗΤΙΚΟΥΣ ΔΙΑΓΩΝΙΣΜΟΥΣ 201 - 300

http://www.mathematica.gr/forum/viewtopic.php?f=109&t=15584

Επιμέλεια: xr.tsif Σελίδα 68

ΘΕΜΑ 271 (ΔΗΜΗΤΡΗΣ ΙΩΑΝΝΟΥ )

Ποιο είναι το τελευταίο ψηφίο του αριθμού: 999999999A 99999 .

(Για την Β και την Γ Γυμνασίου)

Λύση:

Θέλουμε να βρούμε το τελευταίο ψηφίο του αριθμού 999999999A 99999

Επειδή ο αριθμός 99 είναι περιττός, θα έχουμε ότι 99 2k 1 με k φυσικό αριθμό.

Τότε 99 2k 1 2k 2 k999 999 999 999 (999 ) 999

.

Όμως ο αριθμός 2999 λήγει σε 1 . Άρα και ο αριθμός 2(999 )k θα λήγει και αυτός σε

1 και συνεπώς ο 2 k(999 ) 999 θα λήγει σε 9 . Άρα θα είναι περιττός και θα έχει την

μορφή 2n 1 με n φυσικό αριθμό.

Άρα θα έχουμε: 99999 2n 1 2 n

9999 9999 (9999 ) 9999 . Βλέπουμε ότι και αυτός ο

αριθμός λήγει σε 9 και άρα είναι περιττός. Άρα θα γράφεται στην μορφή 2m 1 με

m φυσικό αριθμό.

Άρα θα έχουμε: 2m 1 2 mA 99999 (99999 ) 99999

.

Και από εδώ φαίνεται ότι ο αριθμός αυτός λήγει σε9 .

ΘΕΜΑ 272 (ΔΗΜΗΤΡΗΣ ΙΩΑΝΝΟΥ )

Να εξετάσετε αν ο αριθμός 1968 78

7 1968 3 68A

2011 2001

είναι ακέραιος.

(Για την Β και την Γ Γυμνασίου)

Page 69: ΠΡΟΤΕΙΝΟΜΕΝΕΣ ΑΣΚΗΣΕΙΣ ΓΙΑ ΜΑΘΗΤΙΚΟΥΣ ΔΙΑΓΩΝΙΣΜΟΥΣ 201 - 300

http://www.mathematica.gr/forum/viewtopic.php?f=109&t=15584

Επιμέλεια: xr.tsif Σελίδα 69

Λύση:

Παρατηρούμε ότι 1968 78 1968 78

7 1968 3 68 7 1968 3 68A

2011 2001 10

.

Αρκεί να βρούμε το τελευταίο ψηφίο του 1968 787 1968 3 68 .

Οι δυνάμεις του 8 τελειώνουν σε: 8,4,2,6 και επαναλαμβάνονται ανά 4 .

Άρα 1968:4 492 και 1968 4 4921968 (1968 ) , το 4

1968 τελειώνει σε 6 και όλες οι

δυνάμεις του 6 τελειώνουν και αυτές σε 6 . Άρα το 19687 1968 θα τελειώνει σε 2 .

Όμοια το 4 19 2 47 18 968 36 (68 )68

. Το 468 τελειώνει σε 6 και όλες οι δυνάμεις του

6 τελειώνουν και αυτές σε 6 , άρα το 7868 τελειώνει σε 6 και το 78

3 68 τελειώνει

σε 8 . Άρα 1968 787 1968 3 68 τελειώνει σε 4 που δεν διαιρείται με το 10 .

Άρα ο αριθμός A δεν είναι ακέραιος.

ΘΕΜΑ 273 (ΔΗΜΗΤΡΗΣ ΙΩΑΝΝΟΥ )

Ισόπλευρο τρίγωνο και τετράγωνο έχουν την ίδια περίμετρο. Μπορεί να έχουν και το

ίδιο εμβαδόν; (Για την Β Γυμνασίου)

Λύση:

Έστω a η πλευρά του τριγώνου και x η πλευρά του τετραγώνου. Τα δύο σχήματα

έχουν την ίδια περίμετρο κι έστω ότι έχουν και το ίδιο εμβαδόν. Τότε το σύστημα

2

2

3a 4x

a 3x

4

πρέπει να έχει λύση στο R . Έτσι έχουμε

Page 70: ΠΡΟΤΕΙΝΟΜΕΝΕΣ ΑΣΚΗΣΕΙΣ ΓΙΑ ΜΑΘΗΤΙΚΟΥΣ ΔΙΑΓΩΝΙΣΜΟΥΣ 201 - 300

http://www.mathematica.gr/forum/viewtopic.php?f=109&t=15584

Επιμέλεια: xr.tsif Σελίδα 70

222 222

22

3a3a 3a3a 4x xx x

44 4a 0a 3

9aa 3 9aa 3 3axa 3( )4

44 164 4

. Άτοπο

Άρα δεν μπορεί να έχουν και το ίδιο εμβαδόν.

ΘΕΜΑ 274 (ΔΗΜΗΤΡΗΣ ΙΩΑΝΝΟΥ )

Αν 1 1 1 1

a b c k,a b c k

τότε ένας τουλάχιστον από τους a,b,c θα είναι

ίσος με k , (όπου οι αριθμοί a,b,c,k 0 ). (Για την Γ Γυμνασίου)

Λύση:

a b c kab bc ac 1

(ab bc ac)(a b c) abc1 1 1 1abc a b c

a b c k

2 2 2 2 2 2a b abc a c ab b c abc abc bc ac abc

2 2 2 2 2 2a b abc a c ab b c abc bc ac 0

2a (b c) ab(b c) bc(b c) ac(b c) 0

2(b c)(a ab bc ac) 0 (a c)(a b)(b c) 0

a c 0 a b 0 b c 0 . Άρα b k c k a k .

ΘΕΜΑ 275 (ΔΗΜΗΤΡΗΣ ΙΩΑΝΝΟΥ )

Να βρείτε την ελάχιστη και την μέγιστη τιμή της παράστασης: A 6x 8y αν

γνωρίζουμε ότι 2 2x y 1, x,y R . (Για την Γ Γυμνασίου)

Page 71: ΠΡΟΤΕΙΝΟΜΕΝΕΣ ΑΣΚΗΣΕΙΣ ΓΙΑ ΜΑΘΗΤΙΚΟΥΣ ΔΙΑΓΩΝΙΣΜΟΥΣ 201 - 300

http://www.mathematica.gr/forum/viewtopic.php?f=109&t=15584

Επιμέλεια: xr.tsif Σελίδα 71

Λύση:

2 2A 6x 8y 8y A 6x (8y A) 36x (1).

(1)362 2 2 2 2 2

x y 1 36x 36y 36 (8y A) 36y 36

2 2100y 16Ay A 36 0 , για κάθε y R .

Άρα η εξίσωση έχει λύση για κάθε y R και άρα

2 2Δ 0 144A 14400 0 A 100 A 10 10 A 10 .

Άρα η ελάχιστη τιμή της παράστασης είναι το 10 και η μέγιστη τιμή το 10 .

ΘΕΜΑ 276 (ΔΗΜΗΤΡΗΣ ΙΩΑΝΝΟΥ )

Να αποδείξετε ότι ο αριθμός n n n156 12 13 1 διαιρείται με το 132 για κάθε n

φυσικό αριθμό. (Για την Γ Γυμνασίου)

Λύση:

Το 156 αναλύεται σε γινόμενο παραγόντων: 156 2 2 3 13 12 13 .Έτσι

n n n n n n n n n n156 12 13 1 (12 13) 12 13 1 12 13 12 13 1

n n n n n12 (13 1) (13 1) (12 1) (13 1) πολ12 πολ11

πολ(12 11) πολ132 , για κάθε n φυσικό αριθμό.

ΘΕΜΑ 277 (ΔΗΜΗΤΡΗΣ ΙΩΑΝΝΟΥ )

Αν ο ακέραιος a γράφεται ως άθροισμα τετραγώνων δύο ακεραίων, τότε και ο

αριθμός 10a γράφεται επίσης ως άθροισμα τετραγώνων δύο ακεραίων.

(Για την Γ Γυμνασίου)

Page 72: ΠΡΟΤΕΙΝΟΜΕΝΕΣ ΑΣΚΗΣΕΙΣ ΓΙΑ ΜΑΘΗΤΙΚΟΥΣ ΔΙΑΓΩΝΙΣΜΟΥΣ 201 - 300

http://www.mathematica.gr/forum/viewtopic.php?f=109&t=15584

Επιμέλεια: xr.tsif Σελίδα 72

Λύση:

Έστω 2 2a x y , το 2 2

10 3 1 άρα από την ταυτότητα του Lagrange έχουμε:

2 2 2 2 2 2 2 210a (x y )(3 1 ) (3x 1y) (1x 3y) (3x y) (x 3y) .

ΘΕΜΑ 278 (ΔΗΜΗΤΡΗΣ ΙΩΑΝΝΟΥ )

Αν οι πραγματικοί αριθμοί x,y είναι καθένας τους άθροισμα δύο τετραγώνων, τότε

και ο xy είναι επίσης άθροισμα δύο τετραγώνων. (Για την Γ Γυμνασίου)

Λύση:

Έστω 2 2x a b και 2 2

y c d . Από την ταυτότητα του Lagrange έχουμε:

2 2 2 2 2 2xy (a b )(c d ) (ac bd) (ad bc) .

ΘΕΜΑ 279 (ΔΗΜΗΤΡΗΣ ΙΩΑΝΝΟΥ )

Να αποδείξετε ότι η παράσταση n 2 n 2 n nA 3 2 3 2

διαιρείται με το 10 για

κάθε n θετικό ακέραιο. (Για την Β Γυμνασίου)

Λύση:

n 2 n 2 n n n 2 n 2 n n n nA 3 2 3 2 3 (3 1) 2 (2 1) 3 ·10 2 5 2·5·3 2 ·5

n n 1 n n 12·5(3 2 ) 10(3 2 )

.

ΘΕΜΑ 280 (ΔΗΜΗΤΡΗΣ ΙΩΑΝΝΟΥ )

Μπορεί το άθροισμα πέντε διαδοχικών φυσικών αριθμών να είναι πρώτος;

Το ίδιο να εξετάσετε και για το άθροισμα των τετραγώνων πέντε διαδοχικών

φυσικών αριθμών. (Για την Γ Γυμνασίου)

Page 73: ΠΡΟΤΕΙΝΟΜΕΝΕΣ ΑΣΚΗΣΕΙΣ ΓΙΑ ΜΑΘΗΤΙΚΟΥΣ ΔΙΑΓΩΝΙΣΜΟΥΣ 201 - 300

http://www.mathematica.gr/forum/viewtopic.php?f=109&t=15584

Επιμέλεια: xr.tsif Σελίδα 73

Λύση:

Έστω a , a 1 , a 2 , a 3 , a 4 οι αριθμοί με a N . Τότε

a a 1 a 2 a 3 a 4 5a 10 5(a 2) . Για να είναι το άθροισμα πρώτος

πρέπει a 2 1 a 1 , άτοπο.

Όμοια για το άθροισμα τετραγώνων,

2 2 2 2 2 2 2a (a 1) (a 2) (a 3) (a 4) 5a 20a 30 5(a 4a 4) .

Για να είναι το άθροισμα πρώτος πρέπει 2 2

a 4a 4 1 a 4a 3 0 a 1 a 3 , άτοπο.

ΘΕΜΑ 281 (ΔΗΜΗΤΡΗΣ ΙΩΑΝΝΟΥ )

Να αποδείξετε ότι δεν υπάρχουν ακέραιοι a,b,c,d τέτοιοι ώστε να ικανοποιούν όλες

τις παρακάτω ισότητες:

abcd a 111...1 (το πλήθος των άσων είναι 2011)

abcd b 111...1 (το πλήθος των άσων είναι 2011)

abcd c 111...1 (το πλήθος των άσων είναι 2011)

abcd d 111...1 (το πλήθος των άσων είναι 2011)

(Για την Β και την Γ Γυμνασίου)

Λύση:

Έστω ότι υπάρχουν ακέραιοι a,b,c,d τέτοιοι ώστε να ικανοποιούν όλες τις

παραπάνω ισότητες. Τότε έχουμε:

( )abcd a 111...1

abcd b 111.b a

..1

(1) και

( )abcd c 111...1

abcd d 111.c d

..1

(2) .

Page 74: ΠΡΟΤΕΙΝΟΜΕΝΕΣ ΑΣΚΗΣΕΙΣ ΓΙΑ ΜΑΘΗΤΙΚΟΥΣ ΔΙΑΓΩΝΙΣΜΟΥΣ 201 - 300

http://www.mathematica.gr/forum/viewtopic.php?f=109&t=15584

Επιμέλεια: xr.tsif Σελίδα 74

Από (1) και (2) προκύπτει 2 2 2a c a 111...1 a(ac 1) 111...1 (2011 άσοι) άρα

a 1 και 2 2ac 1 111...1 c 1 111...1 (c 1)(c 1) 111...1

c 1 1 απορρίπτεται και c 1 111...1 c 111...10 (2010 άσοι)

Τότε 2 2 2 2a c a 111...1 (11...10) 1 11...1 (11...10) 11...12 Άτοπο.

ΘΕΜΑ 282 (ΔΗΜΗΤΡΗΣ ΙΩΑΝΝΟΥ )

Αν 1 2 n

a ,a ,...,a είναι θετικοί αριθμοί και έχουν γινόμενο ίσο με μονάδα, να

αποδείξετε ότι n

1 2 n(1 a )(1 a )...(1 a ) 2 . (Για την Γ Γυμνασίου)

Λύση:

Από την ανισότητα x y 2 xy έχουμε

n n

1 2 n 1 2 n 1 2 n(1 a )(1 a )...(1 a ) 2 a 2 a ... 2 a 2 a a a 2 .

ΘΕΜΑ 283 (ΔΗΜΗΤΡΗΣ ΙΩΑΝΝΟΥ )

Οι εξωτερικές γωνίες ενός τριγώνου ABC είναι ανάλογες προς τους αριθμούς 2,3,4 .

Να υπολογισθούν οι εσωτερικές γωνίες του. (Για την Β και Γ Γυμνασίου)

Λύση:

Οι εξωτερικές γωνίες του τριγώνου είναι οι o o o

180 A , 180 B , 180 C

.

Ισχύει

o o o o o o o

o180 A 180 B 180 C 180 A 180 B 180 C 36040

2 3 4 2 3 4 9

.

Άρα o o o

A 100 ,B 60 ,C 20 .

Page 75: ΠΡΟΤΕΙΝΟΜΕΝΕΣ ΑΣΚΗΣΕΙΣ ΓΙΑ ΜΑΘΗΤΙΚΟΥΣ ΔΙΑΓΩΝΙΣΜΟΥΣ 201 - 300

http://www.mathematica.gr/forum/viewtopic.php?f=109&t=15584

Επιμέλεια: xr.tsif Σελίδα 75

ΘΕΜΑ 284 (ΔΗΜΗΤΡΗΣ ΙΩΑΝΝΟΥ )

Να βρεθούν τα δύο τελευταία ψηφία του αριθμού 702 .

(Για την Β και Γ Γυμνασίου)

Λύση:

Είναι 92 512 12mod100 , άρα 63 7

2 12 mod100 και επομένως

70 72 24 mod100 (I).

Όμως, 224 576 24mod100 άρα 7

24 24mod100 (II).

Από (Ι) και (ΙΙ) προκύπτει ότι τα δύο τελευταία ψηφία του 702 είναι 24 .

Β τρόπος

Έχουμε 102 1025 1 25k 1 για κάποιο ακέραιο k . Άρα

70 7 72 (25k 1) 25k ( 1) 25 1 για κάποιο ακέραιο .

Πρέπει να υπάρχει ακέραιος r ώστε 4r 1 ή 4r 2 ή 4r 3 ή 4r 4

οπότε παίρνουμε 702 100r 24 ή 70

2 100r 49 ή 702 100r 74 ή

702 100r 99 . Όμως ο 70

2 είναι πολλαπλάσιο του 4 και από τους πιο πάνω

αριθμούς μόνο ο 100r 24 είναι πολλαπλάσιο του 4 . Επομένως ο 70

2 λήγει σε 24 .

Γ τρόπος

Ας βρούμε το τελευταίο ψηφίο του 70

2 . Είναι 70 17

2 16 4 . Ο αριθμός 17

16 τελειώνει

σε 6 αφού αν πολλαπλασιάσουμε δύο αριθμούς που τελειώνουν σε 6 προκύπτει

αριθμός που τελειώνει σε 6 . Άρα ο 70

2 τελειώνει σε 4 .

Ας βρούμε τώρα το προτελευταίο ψηφίο. Αφαιρούμε το 4 από το 70

2 και διαιρούμε

με το 10 . Το τελευταίο ψηφίο αυτού του αριθμού είναι το προτελευταίο ψηφίο του

702 . Έχουμε

70 34 17

16 152 4 2(4 1) 2(16 1)2·3·(16 16 ... 16 1)

10 5 5

.

Όλες οι δυνάμεις του 16 τελειώνουν σε 6 .

Page 76: ΠΡΟΤΕΙΝΟΜΕΝΕΣ ΑΣΚΗΣΕΙΣ ΓΙΑ ΜΑΘΗΤΙΚΟΥΣ ΔΙΑΓΩΝΙΣΜΟΥΣ 201 - 300

http://www.mathematica.gr/forum/viewtopic.php?f=109&t=15584

Επιμέλεια: xr.tsif Σελίδα 76

Άρα ο αριθμός 16 152·3·(16 16 ... 16 1) τελειώνει σε 2 αφού ο

2·3· 16·6 1 6·97 τελειώνει σε 2 .

Επομένως ο 702 τελειώνει σε 70 10 7 7 7 7

2 (2 ) 1024 (1000 24) 1000k 2424 .

Δ τρόπος

Έχω 70 10 7 7 7 72 (2 ) 1024 (1000 24) 1000k 24 .

Αρκεί λοιπόν να βρούμε τα δύο τελευταία ψηφία του 724 .

Έχουμε

7 7 6 5 4 3 224 24 24 24 24(24 1) 24 24(24 1)(24 24 24 24 24 1) 24

24 25 m 24 6 4 25 m 24 600m 24 πολ100 24 .

Άρα ο αριθμός 724 λήγει σε 24 και άρα σε 24 λήγει και ο δοσμένος αριθμός.

Σημείωση:

Να προσθέσω ότι ο 24 είναι ο μικρότερος αριθμός που αν υψωθεί σε οποιαδήποτε

περιττή δύναμη το τελευταίο διψήφιο τμήμα του μένει αναλλοίωτο δηλαδή 24 .

Άλλοι αριθμοί είναι 25 (για κάθε δύναμη) , 49 , 51 , 75 (για κάθε δύναμη), 76 , 99 .

ΘΕΜΑ 285 (ΔΗΜΗΤΡΗΣ ΙΩΑΝΝΟΥ )

Θεωρούμε τρίγωνο ABC με AB AC . Πάνω στην ημιευθεία AC παίρνουμε

σημείο T τέτοιο ώστε AT AB και πάνω στην ημιευθεία AB παίρνουμε σημείο P

ώστε AC AP . Έστω I το σημείο τομής των ευθειών BC,TP . Να αποδείξετε ότι η

AI είναι διχοτόμος της γωνίας A

. (Για την Γ Γυμνασίου)

Λύση:

Τα τρίγωνα ABC και ATP

είναι ίσα , γιατί έχουν AT AB , AC AP και A

κοινή. Άρα P C

και B T

.

Page 77: ΠΡΟΤΕΙΝΟΜΕΝΕΣ ΑΣΚΗΣΕΙΣ ΓΙΑ ΜΑΘΗΤΙΚΟΥΣ ΔΙΑΓΩΝΙΣΜΟΥΣ 201 - 300

http://www.mathematica.gr/forum/viewtopic.php?f=109&t=15584

Επιμέλεια: xr.tsif Σελίδα 77

Όμοια τα τρίγωνα BPI και TCI

είναι ίσα και άρα τα API και

ACI είναι ίσα γιατί έχουν PI IC ,

AC AP και P C

. Άρα ω φ

.

ΘΕΜΑ 286 (ΔΗΜΗΤΡΗΣ ΙΩΑΝΝΟΥ )

Η αρχή του Περιστερώνα, είναι πολύ χρήσιμη σε αρκετές ασκήσεις.

Έστω 99 ακέραιοι αριθμοί 1 2 99

a ,a ,...,a . Να αποδείξετε ότι υπάρχουν ακέραιοι k,m

με 0 k m 99 , ώστε το άθροισμα k 1 k 2 λ

a a .. π 99. λa ο

.

Λύση:

Θεωρούμε τα 99 αθροίσματα 1 1 2 1 2 3 1 2 99

a ,a a ,a a a ,...,a a ... a .

Αν ένα τουλάχιστον από αυτά είναι πολλαπλάσιο του 99 , τότε έχουμε τελειώσει

Υποθέτουμε ότι κανένα από αυτά δεν διαιρείται ακριβώς με το 99 . Τότε το εκάστοτε

υπόλοιπο θα παίρνει από τις τιμές 1,2,3,...,98 . Άρα τα 99 αθροίσματα μπαίνουν σε

98 θέσεις. Από την αρχή του περιστερώνα, τουλάχιστον δύο από τα αθροίσματα

αυτά δίνουν το ίδιο υπόλοιπο. Δηλαδή υπάρχουν ακέραιοι k,m ώστε

1 2 ka a ... a 99a b και

1 2 ma a ... a 99c b όπου k m .

Αφαιρούμε κατά μέλη τις δύο αυτές ισότητες και έχουμε:

k 1 k 2 ma a ... a 99(c a)

.

Page 78: ΠΡΟΤΕΙΝΟΜΕΝΕΣ ΑΣΚΗΣΕΙΣ ΓΙΑ ΜΑΘΗΤΙΚΟΥΣ ΔΙΑΓΩΝΙΣΜΟΥΣ 201 - 300

http://www.mathematica.gr/forum/viewtopic.php?f=109&t=15584

Επιμέλεια: xr.tsif Σελίδα 78

ΘΕΜΑ 287 (ΔΗΜΗΤΡΗΣ ΙΩΑΝΝΟΥ )

Αν a,b θετικοί αριθμοί και m ακέραιος, να αποδείξετε ότι:

m m

m 1a b1 1 2

b a

.

Λύση:

Μόλις χθες συζητήθηκε εδώ

Β τρόπος

Με την προϋπόθεση ότι ο m είναι θετικός ακέραιος

Έχουμε ότι 2 2(kx z) (nx v) 0 για κάθε k,z,n, v 0 .

Άρα 2 2 2 2 2(k n )x 2(kz nv)x z v 0 .

Άρα πρέπει 2 2Δ 0 (kv) (zn) 2(kv)(zn) .

Με την προϋπόθεση ότι kv,zn 0 , θέτοντας στην πιο πάνω σχέση m m2 2

a bkv (1 ) , zn (1 )

b a , έχουμε:

m m mm m

2 2 21 b a b a b1 1 2 1 1 2 2

b a b a b a

mm

m 1222 2 2 2 4 2 .

ΣΗΜΕΙΩΣΗ:

Χρησιμοποιήσαμε την γνωστή ανισότητα: a b

2b a για κάθε a,b 0 .

Page 79: ΠΡΟΤΕΙΝΟΜΕΝΕΣ ΑΣΚΗΣΕΙΣ ΓΙΑ ΜΑΘΗΤΙΚΟΥΣ ΔΙΑΓΩΝΙΣΜΟΥΣ 201 - 300

http://www.mathematica.gr/forum/viewtopic.php?f=109&t=15584

Επιμέλεια: xr.tsif Σελίδα 79

ΘΕΜΑ 288 (socratis lyras )

Δίνονται οι θετικοί αριθμοί a,b,c με abc 1 . Να αποδείξετε ότι:

68

a b c a b b c c a729

.

Λύση:

Παίρνουμε την τρίτη ρίζα στα 2 μέλη έτσι η ανισότητα γίνεται: 1

2

1 1

3 3 32

(a b c) (a b) (b c) (c a)9

.

Από την ΑΜ – ΓΜ το δεύτερο μέλος είναι μικρότερο ή ίσο με 2

(a b c)3

.Με

αυτόν τον μετασχηματισμό και μετά τις πράξεις η ανισότητα γράφεται

1(a b c) 1

3 που αποδεικνύεται με χρήση της ΑΜ – ΓΜ πάλι.

Η ισότητα ισχύει για a b c 1 .

Β τρόπος

Και μια λύση με κατασκευαστικό τρόπο.

3 6 3

a b c a b c a b c a b b c c a1 · ·

3 3 3 2 2 2

οι παραπάνω είναι εφαρμογές της AM – GM.

ΘΕΜΑ 289 (socratis lyras )

Αν a,b,c θετικοί πραγματικοί αριθμοί με a b c 1 να αποδείξετε ότι:

2 2 21 a b c 2 3abc .

Page 80: ΠΡΟΤΕΙΝΟΜΕΝΕΣ ΑΣΚΗΣΕΙΣ ΓΙΑ ΜΑΘΗΤΙΚΟΥΣ ΔΙΑΓΩΝΙΣΜΟΥΣ 201 - 300

http://www.mathematica.gr/forum/viewtopic.php?f=109&t=15584

Επιμέλεια: xr.tsif Σελίδα 80

Λύση:

2 2 2 2 2 2 2(a b c) a b c 2(ab bc ca) a b c 2 3abc(a b c) ...

ΘΕΜΑ 290 (ΔΗΜΗΤΡΗΣ ΙΩΑΝΝΟΥ )

Να αποδείξετε ότι ο αριθμός 2222 11119589 6051 διαιρείται ακριβώς με το 17 .

Λύση:

Παρατηρώ ότι: 9589 17 564 1 και 6051 355 17 16 .

Άρα o 9589 είναι ισουπόλοιπος με 1mod17 .

Άρα ο 22229589 είναι ισουπόλοιπος με 1mod17 (1). Επίσης ο 6051 είναι

ισουπόλοιπος με 1mod17 .

Άρα ο 11116051 είναι ισουπόλοιπος με 1mod17 (2).

Mε πρόσθεση των (1) και (2) προκύπτει το ζητούμενο.

ΘΕΜΑ 291 (ΔΗΜΗΤΡΗΣ ΙΩΑΝΝΟΥ )

Να αποδειχθεί ότι για κάθε x N , ο αριθμός 23x 1991 είναι άρρητος.

Λύση:

Η υπόριζη ποσότητα είναι 2mod3 , άρα μη τέλειο τετράγωνο, όπως θέλαμε.

Β τρόπος

Έστω ότι ο αριθμός αυτός είναι ρητός. Τότε θα ισχύει: 2 23x 1991 y .

Από την παραπάνω σχέση παίρνω ότι ο 2y είναι ισουπόλοιπος με 2mod3 το οποίο

είναι άτοπο.

Page 81: ΠΡΟΤΕΙΝΟΜΕΝΕΣ ΑΣΚΗΣΕΙΣ ΓΙΑ ΜΑΘΗΤΙΚΟΥΣ ΔΙΑΓΩΝΙΣΜΟΥΣ 201 - 300

http://www.mathematica.gr/forum/viewtopic.php?f=109&t=15584

Επιμέλεια: xr.tsif Σελίδα 81

Γ τρόπος

Έστω 23x 1991 y,y N . Τότε 2 2

3x 1991 y .

Αλλά y 3k ή y 3k 1 ή y 3k 2 .

1η ΠΕΡΙΠΤΩΣΗ: y 3k .

Τότε έχουμε 2 2 2 23x 1991 9k 3(3k x ) 1991 . Άρα θα πρέπει ο 3 να διαιρεί

τον 1991 πράγμα που είναι άτοπο.

Ακριβώς με τον ίδιο τρόπο, καταλήγουμε σε άτοπο και από τις δύο άλλες

περιπτώσεις.

Άρα ο δοσμένος αριθμός είναι άρρητος.

ΘΕΜΑ 292 (ΔΗΜΗΤΡΗΣ ΙΩΑΝΝΟΥ )

Αν x R και 1 2 3 4

a ,a ,a ,a είναι διαδοχικοί ακέραιοι αριθμοί και αν

1 2 3 4(x a )(x a )(x a )(x a ) k , να αποδείξετε ότι k 1 .

(Την άσκηση αυτή, είχε προτείνει το 1989 ο Αχιλλέας Συννεφακόπουλος, μαθητής

τότε της Β Λυκείου, στο περιοδικό "ΕΥΚΛΕΙΔΗΣ Β", μαζί με άλλες αξιόλογες

ασκήσεις)

Λύση:

Οι αριθμοί στο α' μέλος είναι τέσσερις διαδοχικοί άρα αν προσθέσουμε ένα

προκύπτει τέλειο τετράγωνο, έτσι: 2y k 1 .Όπου y το τέλειο τετράγωνο που

προκύπτει. Ξέρουμε επίσης ότι τα τέλεια τετράγωνα είναι μεγαλύτερα ή ίσα του 0 ,

άρα k 1 0 , άρα k 1 .

Β τρόπος

Αν 1

a m , επειδή οι 1 2 3 4

a ,a ,a ,a είναι διαδοχικοί ακέραιοι, είναι

Page 82: ΠΡΟΤΕΙΝΟΜΕΝΕΣ ΑΣΚΗΣΕΙΣ ΓΙΑ ΜΑΘΗΤΙΚΟΥΣ ΔΙΑΓΩΝΙΣΜΟΥΣ 201 - 300

http://www.mathematica.gr/forum/viewtopic.php?f=109&t=15584

Επιμέλεια: xr.tsif Σελίδα 82

1 2 3 4(x a )(x a )(x a )(x a ) (x m)(x m 1)(x m 2)(x m 3)

n(n 1)(n 2)(n 3) όπου n x m .

Τότε, το ζητούμενο προκύπτει άμεσα, από την ταυτότητα

2n(n 1)(n 2)(n 3) 1 [n(n 3) 1] .

ΘΕΜΑ 293 (Socrates )

Έστω A μη κενό υποσύνολο του έτσι ώστε: αν x,y και x y A τότε

xy A . Να αποδείξετε ότι A .

Λύση:

Έστω a ένα στοιχείο του A .Τότε προφανώς για κάθε t , t(a t) A . Άρα

2 3 2f (t) t a t t t[t(a t) t] A . Αλλά επειδή η f είναι πολυώνυμο 3ου

βαθμού, Imf ,άρα A .

Β τρόπος

'Έστω a ένα στοιχείο του A . Αν 2

ax

4 και

2a a 4x

y2

, τότε y(a y) x .

Αφού y (a y) a A , από τη δοθείσα ιδιότητα έπεται ότι x A . Συνεπώς,

2a

( , ] A4

(1).

Αρκεί, λοιπόν, να δείξουμε ότι το A περιέχει όλους τους φυσικούς αριθμούς.

Πράγματι, με

Με x 1 και y a 1 , έπεται ότι a 1 1 (a 1) A (*).

Με x 1 και y a , έπεται από (*) και τη δοθείσα ιδιότητα ότι a 1 a A (**)

Page 83: ΠΡΟΤΕΙΝΟΜΕΝΕΣ ΑΣΚΗΣΕΙΣ ΓΙΑ ΜΑΘΗΤΙΚΟΥΣ ΔΙΑΓΩΝΙΣΜΟΥΣ 201 - 300

http://www.mathematica.gr/forum/viewtopic.php?f=109&t=15584

Επιμέλεια: xr.tsif Σελίδα 83

*****************************************************

Από την (1) και τη (**) το πρόβλημα λύθηκε, αφού έπεται ότι 2

a[ , ) A

4 που σε

συνδυασμό με την (1) μας δίνει A .

*****************************************************

Με x 1 και y a 1 , έπεται ότι a 1 1·(a 1) A , που σε συνδυασμό με

την (**) μας δίνει ότι a 1 A (2).

Αφού 0 A (π.χ. από (1)), χρησιμοποιώντας τη (2) επαγωγικά δείχνουμε εύκολα ότι

A .

Η απόδειξη ολοκληρώθηκε.

Γ τρόπος

x A x 0 A 0 A (1) ,

(x 1) 1 A x 1 A (2) ,

1 ( 1) 0 A 1 A (3) .

(2) x A , άρα από την (3) έχουμε 1 A (4).

Έστω 2 33 33 3 3

0 0 0 0 0 0 0x x x 1 1 A ( x x ) A x x A . Και η

απόδειξη τελείωσε.

ΘΕΜΑ 294 (Socrates )

Βρείτε όλους τους θετικούς ακεραίους n τέτοιους ώστε: d , d / n d 1 / n 1 .

Λύση:

Ισχύουν: n kd και n 1 (d 1) τότε d(k ) 1 (k ή ίσον όταν n d )

k a με a 0 ή ίσον όταν n d .

Page 84: ΠΡΟΤΕΙΝΟΜΕΝΕΣ ΑΣΚΗΣΕΙΣ ΓΙΑ ΜΑΘΗΤΙΚΟΥΣ ΔΙΑΓΩΝΙΣΜΟΥΣ 201 - 300

http://www.mathematica.gr/forum/viewtopic.php?f=109&t=15584

Επιμέλεια: xr.tsif Σελίδα 84

Τότε da 1 , k da a 1 και n d(ad a 1) .

Άρα 2(n,d) (d a da d,d) με a 0 ή ίσον όταν n d .

ΘΕΜΑ 295 (Socrates )

Να δείξετε ότι υπάρχει n N τέτοιος ώστε 1 1 1 1 n

... 201121 2 3 n

.

Λύση:

Θα δείξουμε επαγωγικά ότι 1 1 1 1

... n1 2 3 n . Αυτό αρκεί για το

ζητούμενο γιατί τότε η δοθείσα παράσταση είναι n n

n2 2

, οπότε μπορούμε

να πάρουμε 2n 4022 .

Η περίπτωση n 1 είναι άμεση.

Για το επαγωγικό βήμα, έστω 1 1 1 1

... k1 2 3 k . Τότε

1 1 1 1 1 1

... k k 11 2 3 k k 1 k 1

, διότι

1 1k 1 k

k 1 k k 1

, όπως θέλαμε.

ΘΕΜΑ 296 (Socrates )

Θεωρούμε ένα πλήθοςn 1 ατόμων. Κάθε δύο άτομα συνδέονται με αμοιβαία φιλία

ή αμοιβαία έχθρα. Κάθε φίλος φίλου και κάθε εχθρός εχθρού είναι φίλος.

Αν τα άτομα A και B είναι φίλοι – εχθροί τότε αυτό το μετράμε ως 1 φιλία – έχθρα.

Αν μεταξύ των ατόμων αυτών υπάρχει ίσος αριθμός από φιλίες και έχθρες ,να βρείτε

όλες τις δυνατές τιμές του n .

Page 85: ΠΡΟΤΕΙΝΟΜΕΝΕΣ ΑΣΚΗΣΕΙΣ ΓΙΑ ΜΑΘΗΤΙΚΟΥΣ ΔΙΑΓΩΝΙΣΜΟΥΣ 201 - 300

http://www.mathematica.gr/forum/viewtopic.php?f=109&t=15584

Επιμέλεια: xr.tsif Σελίδα 85

Λύση:

Έστω x τυχόν άτομο και έστω A το σύνολο όλων των φίλων του x και B το σύνολο

όλων των εχθρών του x . Από τα δεδομένα του προβλήματος κάθε δύο μέλη του

A' A {x} είναι φίλοι μεταξύ τους (φίλος φίλου = φίλος) και κάθε δυο μέλη του

B είναι φίλοι μεταξύ τους (εχθρός εχθρού = εχθρός). Τέλος αν πάρουμε ένα άτομο

y από το A' και ένα άτομο z από το B τότε αυτοί είναι εχθροί. Πράγματι αυτό

ισχύει εξ ορισμού αν y x . Αν y x τότε δεν μπορεί ο y να είναι φίλος με τον z

αφού τότε ο x ως φίλος του y θα ήταν φίλος και του z , άτοπο.

Αν λοιπόν γράψουμε a | A' | και b | B | τότε a b n , έχουμε ακριβώς a b

2 2

φιλίες και ακριβώς abέχθρες. Άρα 2 2a a b b 2ab και άρα

2 2 2n a b a b 2ab (a b) είναι τέλειο τετράγωνο.

Επίσης αν 2n m είναι τέλειο τετράγωνο, τότε μπορούμε να χωρίσουμε τα άτομα σε

δυο ομάδες μεγέθους 2

(m m)

2

και

2(m m)

2

ώστε σε κάθε ομάδα ξεχωριστά να

είναι όλοι φίλοι μεταξύ τους και μεταξύ των ομάδων να είναι όλοι εχθροί μεταξύ

τους. Αυτό ικανοποιεί τα δεδομένα του προβλήματος.

ΘΕΜΑ 297 (ΔΗΜΗΤΡΗΣ ΙΩΑΝΝΟΥ )

Μια συλλογή διηγημάτων του Α. Παπαδιαμάντη περιέχει 70 διηγήματα, ένα μιας

σελίδας, ένα δύο σελίδων,..., ένα 70 σελίδων και όχι αναγκαστικά με αυτή την

σειρά. Κάθε διήγημα αρχίζει από καινούρια σελίδα και η αρίθμηση των σελίδων του

βιβλίου αρχίζει από την πρώτη σελίδα. Ποιος είναι ο μέγιστος αριθμός διηγημάτων

που αρχίζουν από σελίδα με περιττό αριθμό; ΕΜΕ – Α Λυκείου

Λύση:

Παρατηρούμε ότι αν ένα διήγημα αρχίζει από περιττή(άρτια) σελίδα και έχει άρτιο

αριθμό σελίδων, τότε το επόμενο θα ξεκινά κι αυτό από περιττή(άρτια) σελίδα.

Page 86: ΠΡΟΤΕΙΝΟΜΕΝΕΣ ΑΣΚΗΣΕΙΣ ΓΙΑ ΜΑΘΗΤΙΚΟΥΣ ΔΙΑΓΩΝΙΣΜΟΥΣ 201 - 300

http://www.mathematica.gr/forum/viewtopic.php?f=109&t=15584

Επιμέλεια: xr.tsif Σελίδα 86

Αντίστοιχα, αν ένα διήγημα αρχίζει από περιττή(άρτια) σελίδα και έχει περιττό

αριθμό σελίδων, τότε το επόμενο θα ξεκινά κι αυτό από άρτια(περιττή) σελίδα.

Θα υπάρξουν, λοιπόν, 34 ή 35 αλλαγές του τύπου:

άρτιος περιττός ή περιττός άρτιος (ανάλογα αν το τελευταίο διήγημα έχει

περιττό ή άρτιο αριθμό σελίδων). Επειδή ξεκινάμε από περιττή σελίδα (1η) θα

έχουμε τουλάχιστον 17 του τύπου περιττός άρτιος και άρα τουλάχιστον 17

διηγήματα αρχίζουν από άρτια σελίδα. Συνεπώς, τα διηγήματα που αρχίζουν από

περιττή σελίδα γίνονται μέγιστα όταν δεν έχουμε καθόλου άρτιος άρτιος και τότε

είναι 35 18 53 διηγήματα.

Αυτό είναι δυνατόν, αν π.χ. τα τοποθετήσουμε με τη σειρά 2,4,6,...,70,1,3,...,69 .

ΠΑΡΑΤΗΡΗΣΕΙΣ:

Με αφορμή την άσκηση 284 , όπου δόθηκαν λύσεις και χωρίς την χρήση των mod

είναι χρήσιμα μερικά θεωρητικά στοιχεία, που παραθέτω:

Έστω a,b φυσικοί αριθμοί

(1) από την ταυτότητα n n n 1 n 1 n(a b) α nα b ... nαb b βγάζοντας κοινό

παράγοντα το a έχουμε:

n n 1 n 2 n 1 n n n(a b) a(a na b ... nb ) b (a b) πολa b

.

(2) Για κάθε θετικό ακέραιο n ισχύει ότι n n n 1 n 2 n 3 2 n 1

a b (a b)(a a b a b ... b )

Άρα έχουμε ότι για κάθε θετικό ακέραιο n : n nπολ( )a bb a .

(3) Για κάθε περιττό θετικό ακέραιο n ισχύει ότι n n n 1 n 2 n 3 2 n 1

a b (a b)(a a b a b ... b ) πoλ(a b) .

(4) Για κάθε άρτιο και θετικό ακέραιο n ισχύει ότι n n n 1 n 2 n 3 2 n 1

a b (a b)(a a b a b ... b ) πoλ(a b) .

Page 87: ΠΡΟΤΕΙΝΟΜΕΝΕΣ ΑΣΚΗΣΕΙΣ ΓΙΑ ΜΑΘΗΤΙΚΟΥΣ ΔΙΑΓΩΝΙΣΜΟΥΣ 201 - 300

http://www.mathematica.gr/forum/viewtopic.php?f=109&t=15584

Επιμέλεια: xr.tsif Σελίδα 87

Ας δούμε μερικά παραδείγματα:

ΠΑΡΑΔΕΙΓΜΑ 1: Να βρεθούν τα δύο τελευταία ψηφία του αριθμού nA 4 (326) .

ΛΥΣΗ

n n nA 4 (300 26) 4 (πoλ300 26 ) 4 πoλ300 4 26

n nπoλ1200 4 (25 1) πoλ100 4(πoλ25 1 )

πoλ100 πoλ100 4 πoλ100 4 .

Άρα ο δοσμένος αριθμός λήγει σε 04 .

ΠΑΡΑΔΕΙΓΜΑ 2: Να βρεθούν τα δύο τελευταία ψηφία του αριθμού 2n 1B 524 .

ΛΥΣΗ

2n 1 2n 1 2n 1B (500 24) πoλ500 24 πoλ100 24 24 24

2nπoλ100 24(24 1) 24 πoλ100 24(πoλ(24 1)) 24

πoλ100 24 πoλ25 24 πoλ100 6 4 πoλ25 24

πoλ100 πoλ100 24 πoλ100 24 .

Άρα ο δοσμένος αριθμός λήγει σε 24 .

ΠΑΡΑΔΕΙΓΜΑ 3: Να βρεθούν τα δύο τελευταία ψηφία του αριθμού 100

C 902 .

ΛΥΣΗ

100 100 10 10C (900 2) πoλ900 2 πoλ100 (2 )

10 10πoλ100 1024 πoλ100 (1000 24)

10 10πoλ100 πoλ1000 24 πoλ100 πoλ100 24 24 24

9πoλ100 24(24 1) 24 πoλ100 6 4 πoλ(24 1) 24

Page 88: ΠΡΟΤΕΙΝΟΜΕΝΕΣ ΑΣΚΗΣΕΙΣ ΓΙΑ ΜΑΘΗΤΙΚΟΥΣ ΔΙΑΓΩΝΙΣΜΟΥΣ 201 - 300

http://www.mathematica.gr/forum/viewtopic.php?f=109&t=15584

Επιμέλεια: xr.tsif Σελίδα 88

πoλ100 πoλ100 24 πoλ100 24

Άρα ο δοσμένος αριθμός λήγει σε 76 .

ΠΑΡΑΔΕΙΓΜΑ 4: Να βρεθούν τα δύο τελευταία ψηφία του αριθμού 1689D 6 .

ΛΥΣΗ

1689 3 663 663 663 663(6 ) 216 (200 16) πολ 200 16D 6

4 663 2652 2 2650 10 265πολ100 (2 ) πολ100 2 πολ100 2 2 πολ100 4 (2 )

265 265πολ100 4 1024 πολ100 4 (1000 24)

265 265 2 2πολ100 4 (πολ1000 24 ) πολ100 4 πολ1000 4 (24 24 24 )

2 263 2πολ100 πολ100 4 [24 (24 1) 24 ]

2 2πολ100 4 [24 24 πολ(24 1) 24 ] πολ100 πολ100 4 24

πολ100 2304 . Άρα ο δοσμένος αριθμός λήγει σε 96 .

ΘΕΜΑ 298 (Socrates)

Σε μια μαθητική κατασκήνωση παρατηρήθηκε ότι:

α) Κάθε μαθητής γνώριζε έναν τουλάχιστον άλλο μαθητή. (Θεωρούμε ότι αν ο

μαθητής A γνωρίζει τον B , τότε και ο B γνωρίζει τον A .)

β) Αν δύο μαθητές έχουν τον ίδιο αριθμό γνωστών, τότε δεν έχουν κοινό γνωστό

μαθητή.

Να δείξετε ότι ένας τουλάχιστον από τους μαθητές έχει μόνο ένα γνωστό.

Λύση:

Έστω Aο μαθητής με τον μέγιστο αριθμό γνωστών. Ας υποθέσουμε ότι έχει ακριβώς

k γνωστούς, έστω τους 1 k

B , ,B . Οι 1 k

B , ,B έχουν κοινό γνωστό οπότε πρέπει

ανά δύο να έχουν διαφορετικό αριθμό γνωστών. Όμως κάθε ένας από αυτούς έχει

τουλάχιστον 1 και το πολύ k γνωστούς. Όμως έχουμε ακριβώς k άτομα άρα από

Page 89: ΠΡΟΤΕΙΝΟΜΕΝΕΣ ΑΣΚΗΣΕΙΣ ΓΙΑ ΜΑΘΗΤΙΚΟΥΣ ΔΙΑΓΩΝΙΣΜΟΥΣ 201 - 300

http://www.mathematica.gr/forum/viewtopic.php?f=109&t=15584

Επιμέλεια: xr.tsif Σελίδα 89

την αρχή του περιστερώνα για κάθε 1 i k ακριβώς ένας από τους j

B έχει ακριβώς

i γνωστούς. Ιδιαίτερα, υπάρχει j ώστε ο j

B να έχει ακριβώς ένα γνωστό, όπως

θέλαμε να δείξουμε.

ΘΕΜΑ 299 (Socrates)

Μπορούμε να τοποθετήσουμε στα τετράγωνα μιας σκακιέρας 6x6 αριθμούς από το

σύνολο { 1,0,1} έτσι ώστε σε κάθε γραμμή, στήλη και διαγώνιο το άθροισμα των

αριθμών να είναι διαφορετικό;

Λύση:

Δεν μπορούμε: Το μέγιστο άθροισμα των 6 αριθμών κάθε γραμμής, στήλης ή

διαγωνίου είναι, φυσικά, 1 1 1 1 1 1 6 και το ελάχιστο είναι 6 . Δηλαδή

έχουμε το πολύ 13 δυνατά αθροίσματα, τους αριθμούς 6, 5,...,4,5,6 . Αλλά το

πλήθος των γραμμών, στηλών και διαγωνίων της 6x6 σκακιέρας είναι

6 6 2 14 13 . Άρα δεν μπορεί όλα τα αθροίσματα να είναι διαφορετικά.

ΘΕΜΑ 300 (Socrates)

Οι θετικοί ακέραιοι 1 2 n

a ,a ,...,a ,... ικανοποιούν τη σχέση n 2 n 1 n

a a a 1 για κάθε

θετικό ακέραιο n . Να δείξετε ότι αν n 5 τότε ο αριθμός n

a 7 είναι σύνθετος.

Λύση:

Έχουμε ότι k

a 1 για κάθε k 3 .

Επίσης, για κάθε k 2 έχουμε (k

moda ):

k k 1a 0, a 1

και από τον αναδρομικό τύπο βλέπουμε ότι

k 6 ka 22moda

. Άρα ο

k 6a 22

διαιρείται με τον

ka και, αφού όλοι οι αριθμοί

ka με k 3 είναι

μεγαλύτεροι του 1 , όλοι οι αριθμοί k

a 22 με k 9 είναι σύνθετοι.